116
February 2012 Medicine: 1. 2yrs old child come with parents having rash on arm and trunk and cervical lymphadenopathy for 4 days . WOF is the Dx? a. Vaccination of the child is fully done b. His siblings are fully vaccinated. c. They are having pet in their home. 2. Picture o fundoscopy of retinal vein thrombosis 3. Picture of pyogenic granuloma given…..What is the treatment ( Local excision) 4. ECG given related to Digoxin toxicity 5. 30 yrs old lady travelling from Singapore complicated about cough, fever, no symptoms of shortness of breath given, on examination bilateral basal crackles . What is the Dx? a. Pulmonary embolism b. Chronic Fatigue syndrome. c. Pneumonia d. Tuberculosis 6. Which one of the following drug has ionotropic effect? a. Digoxin b. Digitalis c. Frusemide d. Deltiazem 7. CT scan of man of 74 yrs old given .According to his son he was normal 14 days back and suddenly developed confusion, drowsy . No history of fall given clearly , no other neurological signs and symptoms were given . WOF is Dx? a. Subdural Haematoma. b. Extradural Haemorage. c. Subarachnoid Haemorage. d. Cerebral infarction. 8. History of soccer player injured his Rt Knee .Sudden pain and swelling and locking of Rt knee . WOF is the cause. a. Ant. Cruciate Ligament. b. Lateral Collateral Ligament . c. Tear of medial meniscus. d. Medial collateral ligament.

Feb 2012 AMCQ Recall

Embed Size (px)

DESCRIPTION

AMCQ recall FEB 2012

Citation preview

Page 1: Feb 2012 AMCQ Recall

February 2012Medicine:1. 2yrs old child come with parents having rash on arm and trunk and cervical

lymphadenopathy for 4 days . WOF is the Dx?a. Vaccination of the child is fully done b. His siblings are fully vaccinated.c. They are having pet in their home.

2. Picture o fundoscopy of retinal vein thrombosis3. Picture of pyogenic granuloma given…..What is the treatment ( Local excision)4. ECG given related to Digoxin toxicity

5. 30 yrs old lady travelling from Singapore complicated about cough, fever, no symptoms of shortness of breath given, on examination bilateral basal crackles . What is the Dx?a. Pulmonary embolismb. Chronic Fatigue syndrome. c. Pneumoniad. Tuberculosis

6. Which one of the following drug has ionotropic effect?a. Digoxin b. Digitalisc. Frusemided. Deltiazem

7. CT scan of man of 74 yrs old given .According to his son he was normal 14 days back and suddenly developed confusion, drowsy . No history of fall given clearly , no other neurological signs and symptoms were given . WOF is Dx?a. Subdural Haematoma. b. Extradural Haemorage.c. Subarachnoid Haemorage.d. Cerebral infarction.

8. History of soccer player injured his Rt Knee .Sudden pain and swelling and locking of Rt knee . WOF is the cause.a. Ant. Cruciate Ligament.b. Lateral Collateral Ligament . c. Tear of medial meniscus.d. Medial collateral ligament.

9. Old women with history of shunt of common bile duct , 2 weeks back now complaining of sweeting, dark color urine, no symptoms of fever, Rt Upper quadrant pain given. WOF is the Dx.?a. Pancreatitisb. Cholangitisc. Rupture of bile duct.

10. Picture of man given with SCC of hand. 11. Worker in insulating factory for 20 yrs, with history of smoking for 30 yrs and 15

packs per day, complaints of SOB, 6 Kg Wt loss and haemoptysis. What is the Dx?a. Mesothelioma.b. SCCc. Tuberculosis.

Page 2: Feb 2012 AMCQ Recall

12. All are true for Cluster headache , except:a. Duration less than 2 hoursb. Rhinorrhoea.c. Lacrimationd. Happen 2-3 hours after sleepe. Accompanied by vomiting.

13. Old lady develop sudden back pain after closing window, she is complaining of tenderness of lower back area, after investigation, it was noted that she is having osteopenia. WOF is the cause of her presentation. a. Osteomyelitisb. Multiple myeloma.c. Osteoporosis.d. Stress fracture.

14. 22 yrs old girl had MVA, 5-11 ribs fractured, she is having SOB, tachypnoea,WOF is the most immediate management for her air travel? a. Needle aspiration at 2nd intercostal apace. b. Underwater seal drainage.c. Intubation and ventilation

15. Picture of tinea capitis.16. Picture of Scaphoid fracture given and question about how to apply the cast. 17. Treatment of keratoacanthoma given and treatment asked.18. 35 yrs old man complaining of pain in testes, USG was done and normal. WOF is the

cause?a. Vericoceleb. Epididymitis c. Tortion of Testes

19. 60 yrs female come with history of abdominal distention, constipation and decreased bowel sound , PR was normal, previous H/O appendisectomy at 44 yrs old of age ….X-Ray given …WOF is the cause?a. Sigmoid Volvulusb. Small intestine obstructionc. Ca Caecum.

20. ECG was given. After MI patient develop Arrhythmia, WOF is the drug of choice?a. Digoxinb. Amiodaronec. Atropine

21. 25 yrs old man develop SVT , vagal manuver done , still he was having increase heart rate, he didn’t want adenosine, WOF is next drug to administer?a. Digoxin.b. Atenololc. Verapamil

22. 10 yrs old boy had minor wounds with no past H/O vaccination for tetanus, on examination wound was not contaminated too much. Most appropriate management?a. Igb. DPTc. Tetanus toxoid+Ig.

Page 3: Feb 2012 AMCQ Recall

23. Pic of abdominal mass give protruding out of umbilical area and treatment was asked.a. Debribeb. Surgical intervention.

24. 64 yrs old woman complaining of weakness and fatigue for last few days , there is no family history of any malignant condition . WOF investigation sud be performed?a. Colonoscopyb. FBCc. FOBTd. Sigmoidoscopy

25. 28 yrs old man with a history of URTI , after 1 week there is haematuria, HTN, Proteinuria. WOF is the cause?a. Ig A nephropathyb. Nephrotic Syndrome.c. Renal failured. FSGS.

26. Pt going for cholecystectomy is on warfarin , INR 2.2, how you proceed and adjust Warfarin?a. Delay surgery for 3 daysb. Stop Warfarin and start Heparin and do surgeryc. Stop Warfarin , give FFP and proceed to surgery. d. Continue Warfarin.

27. 46 yrs old woman H/O thyroidectomy few hours ago , nurse noted pt is suddenly having difficulty in respiration, and condition was deteriorating . what is your next step in Mx?a. Call the senior surgeon.b. Shift patient to operation theatre.c. Give her bedside O2.d. Open sutures in ward.

28. 29 yrs old had MVA , tibia fractured , after few hours there is redness and absent pulses at lateral malleolus , what will be your next step investigation. a. Doppler USGb. Arteriography.

29. 62 yrs old lady noticed lump in her breast, after a dog jumped on her chest, WOF is the cause of her lump?a. Fibroadenosisb. Fat necrosisc. Ca Breast.

30. 26 yrs old patient having Diarrhoea and vomiting for last 2 days, O/E drosy and lethargic. WOF is the nest step in Mx?a. Colloids.b. Hartmans solutionc. Crystalloids

Page 4: Feb 2012 AMCQ Recall

31. A 20 yr old presented with weakness and edema of left hand,with h/o cat scratch 4 days ago..lymph nodes enlarged and non tender...what s the cause?

a) cellulitis b)cat scratch c)axillary vein thrombosis

32. patient cannot extend wrist and fingers...all other things normal...lesion...

a)anterior interosseous nerve b)ulnar nerve c)radial n d)posterior interosseous n

33. 23 yr old nurse working n rural hospital presented with c/o cough, foll by hemoptysis for 1 mnth....crr given....(?apical lobe affected) diagnosis

a)hiv b)tuberculosis c)lung ca d)pneumonia

34. bacterial conjuctivitis pic given....treatment

a)symptomatic b)ciprofloxacin drops c)chloramphenicol eye drops

35. man with symptoms of difficulty in plantar flexion,inversion,dorsiflexion,eversion of foot...knee reflex present..where s the lesion.

a)sciatic n b)L5 c)tibial n d)common peroneal n

36. ct scan on cerebral tumor,cerebral h'ge

37. stem on heparin induced thrombocytopenia...wht to do next?he s on low mol wt heparin

a)change to unfractionated heparin b)change to another anticoagulant other than heparin grp c)?????

38. 75 yr old with c/o bloating ,abdo distension, diarrohea alternating with constipation....no blood n stools...colonoscopy done 12 mnths back,,came out norml....wht to do nxt?

a)repeat colonoscopy b)diet advice c)?????????????

Page 5: Feb 2012 AMCQ Recall

39. 75 yr presented with post prandial abdo pain....othr thng norml....diagnosis?

a)Ca duodenum b)Ca cardia c)cholecystitis d)?????????????

40. management of tennis elbow

a)no option for rest b)elbow immobilisation splint c)brace below elbow.

41. typical qs on bells palsy

42. biker fell frm bike 1week back.....no h/o unconsciousness...now presenting with occipital headache....diagnosis??

a)EDH b)SDH c)post traumatic headache

43. ECG of wpw syndrome wth wide qrs...management

a)amiodarone b)adenosine c)cardioversion

44. sign of acute limb ischemia indicative of surgery

a)pain b)paralysis c)color

45. sign of chronic limb ischemia

a)pain b)paralysis c)rubor

46. qs on trichotillomania47. 2 week infant presented with fever,neck stiffness..qs abt meningitis....causative

organism...

a)E.COLI b)strep.pneumonia c)kleibsiella

48. pic of scc.....wht happens next

a)involutes b)grows fast

c)spread to lymph nodes

Page 6: Feb 2012 AMCQ Recall

49. child with petechia and bruises after URTI....diagnosis and inv..........50. Pic of psoriasis Tx.

51. pt on metformin,perindopril,diuretic......amox was tkn fr urti 3 days back....dvlpd swelling and itching of rt side of tongue.....cause fr tht

a)mox b)perindopril c)diuretic

55. indigenous boy brought,,unconscious...cause a)petrol sniffin b)alcohol c)marijuana

52. qs on serotonin syndrome53. treatment of alopecia areata

a)application of hydrocortisone on tht area b)injection of steroid on tht area c)????????

54. qs on conduct disorde and adhd55. mre qs on peptic ulcer ds,,Ca duodenum,,Ca Cardia56. Indigenous boy brought,,unconscious...cause

a)petrol sniffin b)alcohol c)marijuana

57.What is the risk of conceiving a child with Down Syndrome for a 38 year old female. A. 1:10B. 1:100C. 1:200D. 1:400E. 1:700

58. 19 year old boy complains of having difficulties in playing sports at school. His height is at 97th percentile for his age and weight is at 3rd percentile. What is your next step in management -

a) Karyotyping b) bone scanc) growth hormone testd) CT Scane) Cardiac ultrasound

59.A pt who worked in a gold mine for 20 years and is smoker and history of working in petrol station for 20 years now presents with hematemasis and cough. Xray was given, how would diagnosis him

A) Bronchoscope B) USGC) Ct scan D) Needle aspiration

Page 7: Feb 2012 AMCQ Recall

60. A Ct scan of a 80 years old man was given ( massive bleed with displaced ventricles, is admitted to the ICU, pts wife want a calm ending for him while the Son want to do want ever it takes to save his father. Who would decide on the treatment?

a) Follow the wife wishes b) Follow the son wishes c) Decide by your own using the doctor powerd) Arrange family meetinge) Guardian ship court

61. A middle aged lady w hx of pain over distal fingers - more on palpation with no obvious deformity for 20 years + Anaemia + CXR shows enlarged heart .what is the diagnosis?

a) scleroderma b)SLE

62. A picture of psoriasis of the leg with extensive lesions. What is the treatment of choice?a)UVB light b) calcitriol

63. A pic of SCC at lower lip, what is most common cause?a) Smokingb) Sun exposurec) Metsd) Unknown cause

64. A critically ill pt who mad advance directives when he was in good health now no tubesNow he has a massive stroke and critically ill. His family also agreed with his decision previously made. How will you decide on his rx nowa) Intibuate and ventilateb) IV line and feedc) Cvp and fluidd) Oral small sips of watere) Send him for terminal care

65. A pt had an accident his pulse is 120 BP is 120/80 laying and 90/70 standing what is the percentage of blood loss

a) 10 %b) 20 %c) 25%d) 35%e) 40%

66. a 18 month old has head circumference 25th percentile at birth 50th percentile at 6 moth 75th percentile at 1 year and 97th percentile at presentation what is the best way to diagnose.

a) Cranial USGb) MRI c) No need to investigate

67.ECG of SVT from blue book asked diagnosed68. old home pt with off and on constipation and Xray of the sigmoid volvulus on blu book page 458 was given and asked for diagnosis

a) sigmoid volvulusb) lage bowl obstructionc) small bowl obstructiond) adhission

Page 8: Feb 2012 AMCQ Recall

69. ecg of WPW asked for rxa) Carotid messageb) amidrionc) digoxind) adenosine

70. SVT ECG from blue book and dx asked.

71. ECG of an old lady with SVT and then sudden a systole after carotid message . asked for tx

a) amioderonb) adenosinec) shift her to ICUd) atropine i/ve) do not do carotid message again.

72. venous ulcer asked about the most common associationa) smokingb) DVTc) long bone fracture

73. a pic of ulcer o the medial meallolus what the diagnosisa) arterial ulcer b) venous ulcrc) diabetic ulcer

74. Patient complains of jaundice with dark urine and pale stools. Pain in right upper quadrant. There is a mass which moves with respiration. Most likely diagnosis?

a) Choledocolithiasisb) Carcinoma of ampulla of pancreasc) Mucocele

75.An old pt on who just had drug elucidating prosthetic valvein place is on clopidrogril came in to preoperative clinic for evaluation. He has a non complicated vericocele. Wht will you advise him regarding the operation

a) reacess the nedd for durgeryb) stop clopidrogil and shift to heparin one week before surgeryc) give FFP before surgeryd) stop clopidrogril few days before surgery e) giv vit K

76. A 60 year old man with a long history of diabetes, comes to the hospital with ulcer in each dorsum of the toe, in both the legs. There is foul smelling discharge from the right side ulcer. What would be the most appropriate next step in management?

a) Amputate the toes.b) Do Doppler Ultrasoundc) Give antibiotics (Amoxicillin and Clavulonic Acid)

77. A 30 year old man with multiple rib fractures following a motor vehicle accident. Having paradoxical breathing. what would be the most appropriate next step in management?

a) strap the chest wellb) CPAP with Ventilationc)no option for intubation and ventilation

Page 9: Feb 2012 AMCQ Recall

78. pic of red eye with watery discharge , two weeks earlier pt had oral and genital ulcers what the rx

a) antibioticsb) chlormphencol ointment c) oral steroids d)steroid drops

79. Q on leukaemia with blast cells 80. a pt after being drinking and sudden pain in and vomiting in the epigastria area and shock, the pain came after bout of vomiting what is mx.

a) endoscopy b) gastro graffinc) Ct chest

81. a young male came as one of his friends recently has been diagnosed with the testicular cancer. You examined him and found that one testicle is slightly large the other no other abnormality detected. How will you advice him.

a) reassure him as nothing is wrongb) advice him monthly self testicular examinationc) annual AFPd) regular review with USG

82. ct of intra cerebral bleed dx asked83. typical Presentation of ITP initial test asked

1) FBC 2) Apttc) INrd) bleeding time

84.which of the following is true regarding Naltrexone A. Short actingb. Given for chronic marijuana usersc. Ideal for IV drug user since it is long acting.d. A substitute for Methadonee) . Generic name for naloxone

85.A middle aged women with calf pain from few days came to hospital. her investigations showed Deep vein thrombosis. What is the management?

a. IP management with compression with compression socking and LMWHb. IP management with elevation and LMWHc. OP management with compression and LMWHd. OP management with leg elevation and LMWH

86.Middle aged man with history of cough.CXR showed, lower lobe consolidation. What is the management?

a. oral amoxicillin and clavulanateb. oral azithromycinc.IV ampicillind. Oral flucloxacillin

87. All are S/E of corticosteroid therapy except: - hirsutism- ecchimosis- leucocytosis- osteomalasia- lymphopenia

Page 10: Feb 2012 AMCQ Recall

88. IN a patient with GBS which is the appropriate way to monitor his oxygenation?a) Forced vital capacity in 1secb) forced expiratory capacity in 1 secc) blood gasesd) pulse oxymetere) Oxygen saturation

89. A 14 yr old boy separated from his parents n currently living w his father since 3 months having bloody diarrhoea .no fever .what is the best investigation to come to a diagnosis?

a) colonoscopy b) sigmoidoscopy c)celiac serology d)FBC

90. A 2 week old child with temp of 40'c came to hospital w hx of seizure + looking ill. Mother very concerned as child not feeding well. What is the diagnosis?

a) febrile convulsion b) septicaemia c) encephalitis d)epilepsy

91. A man presented with the painful defecation and blood on stools. Pain persist for almost 30 min after he opens bowls. What the MX?

a) DREb) colonoscopyc) high fibre dietd) high fibre diet and glyceraltrinitrate ointment e) Steroid ointment

92. A 12 years old boy h/o Nephrotic syndrome not responding well to high dose cortico-steroid therapy, urine protein-++,hypertension, hematuria with red casts. WOF can be the cause:

A .Ig A nephropathyB .PSGNC .focal glomerulosclerosis

93.You are a medical officer in tertiary hospital, a pt is about to undergo abdominal surgery. Just before the surgery when the pt is being shifted to the operation theatre, the nurse told you that the consent for surgery has not been taken. What you do about the consent now?

a) Send the pt to surgery, as the anaesthetist can do this just before anaesthesiab) Take the consent and send the pt for surgery c) Download information from internet provide it to pt and take the consent.d) Call the operating surgeon and ask him to take the consente) Ask the attending nurse to take the consent from the pt while sending him to OT.

94. What is the characteristic sign of chronic limb ischemiaa ) pain relieved by walkingb) ruborc ) color of limb improved after hanging limb at end of bedd ) numbness

Page 11: Feb 2012 AMCQ Recall

95. a girl is coming to the gp for evaluation with her parents as she is unable to concentrate on her studies. Although she go out with her friends, she dose not seem to enjoy much of the activities, she also experiences some sleep disturbance, which of the following piece of information in the history is most helpful to reach a diagnosis.

a) mother also experience sleep disturbanceb) Brother has ADHDc) She has suicidal ideationd) She use drugs at schoole) She is being bullied at school

96. Bacterial meningitis, viral meningitis, encephalitis please read all about them.97. History of eye injury blowout what will you find in the signs

a) diplopia b) hyphemiac) fracture zygoma

98. SCC of lung and mesothealeoma do these topic

99. Diabetic pt on metformin, endepamide having hypoglycaemic attacks what’s the reason for hypoglycaemia

a) metformin b) endepmide

100. another q on diabetes controlled at the moment but HBA1c increased 7.5 asked the rx, cannot remember the options.101.Q on sodium low 120 asked the reason can’t remember the full Q.

102. Q on intestinal obstruction 3 ecg Question 3 ct scan Q 2 Xray chest4 skin condition with pic

103.23 yr old nurse working n rural hospital presented with c/o cough, foll by hemoptysis for 1 mnth....crr given....(?apical lobe affected) diagnosis

a)hiv b)tuberculosis c)lung ca d)pneumonia

104.bacterial conjuctivitis pic given....treatment

a)symptomatic b)ciprofloxacin drops c)chloramphenicol eye drops

106.ct scan on cerebral tumor,cerebral h'ge107.stem on heparin induced thrombocytopenia...wht to do next?he s on low mol wt heparin

a)change to unfractionated heparin b)change to another anticoagulant other than heparin grp c)?????

Page 12: Feb 2012 AMCQ Recall

108.75 yr old with c/o bloatin,abdo distension,diarrohea alternating with constipation....no blood n stools...colonoscopy done 12 mnths back,,came out norml....wht to do nxt?

a)repeat colonoscopy b)diet advice c)?????????????

109.75 yr presented with post prandial abdo pain....othr thng norml....diagnosis?

a)Ca duodenum b)Ca cardia c)cholecystitis d)?????????????

110.18 weeks pregnant who s a known c/o cin2 with compalins of clear vaginal dischrge....wht u do next....

a)do pap smear now. b)cervical length measurement.. c)?????????

112.typical qs on bells palsy113.management of tennis elbow

a)no option for rest b)elbow immobilisation splint c)brace below elbow

114)biker fell frm bike 1week back.....no h/o unconsciousness...now presenting with occipital headache....diagnosis??

a)EDH b)SDH c)post traumatic headache

115)ECG of wpw syndrome wth wide qrs...management

a)amiodarone b)adenosine c)cardioversion

116.2 week infant presented with fever,neck stiffness..qs abt meningitis....causative organism...

a)E.COLI b)strep.pneumonia c)kleibsiella

117.pic of psoriasis....treatment

Page 13: Feb 2012 AMCQ Recall

118.pic of scc.....wht happens next

a)involutes b)grows fast c)spread to lymph nodes

119.child with petechia and bruises after URTI....diagnosis and inv..........

120.pt on metformin,perindopril,diuretic......amox was tkn fr urti 3 days back....dvlpd swelling and itching of rt side of tongue.....cause fr tht

a)mox b)perindopril c)diuretic

121.indigenous boy brought,,unconscious...cause a)petrol sniffin b)alcohol c)marijuana

122.treatment of alopecia areata

a)application of hydrocortisone on tht area b)injection of steroid on tht area c)????????

123.qs on conduct disorde and adhd124.mre qs on peptic ulcer ds,,Ca duodenum,,Ca Cardia

125..17 years old jogging then collapsed.somebody did CPR and was revived when she arrived at ER. Has history that father died suddemly. Which is the likely diagnosis?

--HOCM

126. ECG on V tach. Identify and Treatment--Amiodarone

127.Woman when she defecates a mass is noted to bulge out of her introitus.Most likely diagnosis.

---Rectocoele128. child with constipation. What is the next management

--laxative129. child with bruises and petechiae after a viral infection weeks ago.most likely diagnosis?

--ITP130.ct scan of rectus sheath hematoma131. situation about testicular sweeling. What do you do next?

--ultrasound--FNAC

132. question about painless hematuria--bladder tumor--renal tumor

Page 14: Feb 2012 AMCQ Recall

133.. question about a man 35years old asking about his risk of developing prostate CA. Father diagnosed prosteate CA at 85 years old.

--reassure?--FOBT--colonoscopy

134. what is the most important sign to indate for immediate embolectomy--paralysis--pain--rubor

135. 6weeks old infant with fever. Mother notes some twitches before. What is the most likely diagnosis

--septiceamia--febrile convulsion

136.old man was found by police and brought to hospital. Daugther has many complaints about father. She said he has been living in squalor and eating rubbish.what kind of dementia?

--frontal lobe dementia--lewy body dementia

137. what to give to pregnant woman aside from folic acid to prevent NTD?--iodine

138. question july 2010 #101139.question july 2010 #110140. question july 2010 #111141. question about an old lady after closing the window (?) sustained a crush fracture on her vertebrae. She had a history of opertation for CA of ceucum

--metastasis from primary cancer--osteoporosis

142. patient just had cricothyroidectomy and developed stridor while she was still in the recovery room. What will you do?

--call the surgeon--explore the wound in the ward--remove the skin sutures

143. xray given. Man is a non-smoker and working in the mine industry (did mention the years, think it was more than 20years). Has been coughing

--mesothelioma

144. question about percentage passing the disease to their children with cystic firbosis-- 0%

145. xray given. Man coughing, smoker and working in the mine industry. What is the most probable diagnosis?

--brochogenic CA

146. a child with fever. What is the indication that she has a severe bacterial infection--unresponvie to mother--pallor

147.. xray of sigmoid volvulus

Page 15: Feb 2012 AMCQ Recall

148. Boy having balck and white drawings. Parent just separted. What is the most appropriate Mx?

--ask the boy about his drawing149. question about domestic violence. Pregnant lady with bruises. What do you do?

-- give the numbers150. woman with 3 children wants space, but in file said “do not give OCP” as Stand by husband

-- its her opinion and give her OCP151.NALOXONE is

-- Short acting

152. 17yo wants OCP. She had seizsure whihc is controlled with carbamepine and subdural haematoma

-- GIVE OCP--GIVE OCP AND USE CONDOM*

153.X-ray of hip osteopenia ALP is 800 ( that is the only information given ) Rxa. Biphosephanateb. Cac. Vita b12d. Radiotherapy

154.Pic of both lower limbs showing red rash. 34y old male with diarrhoea and admitted in the hospital Dx

a. HSP

155.Mallert finger pic. Which type of injury will occura. Hyperflexion occur due to extensionb. Rupture of tendon

156.CXR – night sweat, dry cough, wt loss, enlarge bilateral cerviacal lymphnode Dxa. Sacodosisb. Tbc. Lymphomad. CLL

157.A patient who started hydrochlorothiazide with digoxin develop nausea electrolyte pic given only potassium decrease

a. Stop digoxin and give K supplimentb. Stop hydrochlorothazide and give K supp

158.SVT ECGa) Adenosingb) Amiodaronec) Verapermild) Dogoxin

159.ECG . a pt who had stroke came to hospital on 5# day she collapsed in the toilet. BP pulse?? What will you do next to dx his condition?

a) Ctpab) Echo cardio8. A boy came with bloody diarrhea with less urine output what is the dxa) Renal failureb) HUS

Page 16: Feb 2012 AMCQ Recall

160.A infected batholian cysta) Antibiotic will resolve itb) Cause by gonnorrhoeac) It is asymptomatic

161.A pt who had an injury to eye due to hit from metalic spike what will you doa) Use local anaesthetic and removed

162.A pt 70 yrs old man came to the hospital think that he is having serial illness coming to doctor with different names. What is it

a) Facticious disb) Depressionc) Normal grief

163.Abd x-ray 5th POD due to hip replacementa) Pt rectus tubeb) Gastro graffin enamac) Colonscopy

164.5th POD for input output chart given.165.In both recall output is more than input. One is 1400 is differnt, other recall 2100.

a) Resolved paralytic iliusb) Wrong chart

166.A lady and husband came to you recently dx cystic fibrosis concern abouthe their child getting it

a) 0.25 chanceb) 0.5 chance

167.A couple comes telling that one child diagnosed as cystic fibrosis concern about next child incidence

a) 0.5b) 0.25c) 100

168.You went to old house to trat a person and prescribe b-blocker and asprin. The nurse told that he is not responding to B-blocker what ill you do

a) Talk with nurse durin her breakb) Report nurse managerc) Do as she told

169.You consider to old care notice that it is written in their chart telling dont crash the medicine. But nurse use to crush the medicine to pt as it is hard for him to swallowing what will you do

a) Complain nurse managerb) Go to nurse boardc) Complain to the sup in age care.

170.CT head show hemorrhage. Pt on warfarrin and INR 4.5 what will you do a) giveFFPb) give platelet

171.CT head showing same lesion like infactiona) Multi infarct dementiab) Cerebral hmgc) Cerebral abcess

Page 17: Feb 2012 AMCQ Recall

172.VT ecga) Adenosinb) Amiodaeronec) Adrenaline

173,A lady comeplain loss of finger grip in index and thumb what nerve is having.a) Radial n.b) Carpal tunnelc) Ant. Intoressisisd) Ulner n.

174.80 lady comeing with rectocelea) Pessaryb) Sxc) Pelvic exercise

175. 50 yrs old man coming with sccrotal pain which is reddened what ill you doa) Urine cathab) b/d culturec) FNAC

176. male taken to a research and during it he become dementia and his primary carier is his son what will you do

a) Ask from sonb) Gardianship c) Ethic and comity

177.Fundoscopy pic, HTN ployuria, ploydypsia, what is dxa) DMb) HTNc) CRAOd) CRVO

178.A male who use to work in mine in 20yr smoker 40yr what is the dx. CRX givena) Smallcell carcinomab) Squamous cell carcinomac) Metho

179.A chinese lady come to your. Husband is recenly dx as hemochromatosis having 9 & 18 year kid. Wife is not screen for heamochromtosis. What will you do

a) Check wife onlyb) Check 9 yr onlyc) Check 18 year d) Check both children

178.A down syndrom under go surgery develop hypersensitivea) Latexb) Anaestheic drugs

179.A 34yr male havin, ACEI Statin b-blocker develop calf pain tenderness in calfa) Check ck level creatinb) Check FBC

Page 18: Feb 2012 AMCQ Recall

180.A pic both eye increase tearing who will you doa) Refer plastic surgeryb) Cold compression

181.CXR given and ask of diagnosisa) Pul edemab) Metastasisc) Carcinoma

182.A 2nd POD a person develop agitated confusion. Temp normal and pulse and BP normal what is the cause

a) Alcohol withdrawalb) Pulmonary atelectesiac) Electrolyte imbalance

183.A yong boy comes with rashes in both hand and arthralgia. What will you do to dx ita) CRP and ESRb) Echoc) CXR

184.Coal miner, working since 40 years, no smoking Hx. CXR : pleural effusion. Dx?Infected Bartholin cyst? MxPt with Paget’s disease. What is it associated with?Infected Bartholin cyst. What is true?a. Is asymptomaticb. Treated by a/bc. Caused by gonococcus

185.Old man with Hx of confusion and forgetfulness. He is driving at night. What is your advice?

a. Stop driving

186.AFlatter: full ECG – Dx?

187.Pt with dizziness + palpitation + elevated JPV.5 ECG strips: a. VTb. AFlatterc. SVTNB: no AFib option!

188.23y.o. male c/o tenderness of left scrotum. Transillumination – (-)ive. Next Ix?a. USGb. Urine culturec. Biopsyd. FNA

189.Big stem: old man, with many co-morbidities + polypharmacy, falls every morning.Most appropriate Ix?a. CTb. ECGc. EchoNB: no BSL option!

Page 19: Feb 2012 AMCQ Recall

190.Carpet layer presents with pain and tenderness in one knee. Tx? a. Corticosteroids intra articularb. Rest for 2 wksc. Compression bandage

191.Big stem: 32y.o. female with a lot of complains and Hx. CXR: patchy infiltrates (but looked normal to candidate). Tx?

a. Azithromycin b. Prednisolonec. Clotrimazole d. Ticaracillin

192.Husband is diagnosed with hereditary haemochromatosis. Wife is healthy. They have two children: 9y.o. and 18y.o. Whom are you going to screen?

a. 9y.o.b. 18y.o.c. Both

193.How many children will be affected by Cystic fibrosis, if one of the parents is a career?a. 0b. 0.5c. 0.25

194.Pt complains of developing digital ulcers when exposed to cold in winter. Tx?a. Hydroxychloroquineb. Oral prednisolonec. IM prednisolone

195.Pt agitated and confused, brought in to ED. What will you give?a. Haloperidolb. Olanzepine c. Diazepamd. Fluoxetine

196.Old man living alone, wife died few years ago. Doesn’t want to live. WOF would be of more concern?

a. He is looking for suicidal methods on internetb. His any previous suicide attemptsc. He is living alone

197.Pic of Staphylococcal pneumonia.ECG of VT. Mx?a. Carotid massageb. Amiodaronec. Verapamil

198.Mother brought the child with some problem. Doctor ordered CT. Mother couldn’t bring her child for the appointment. What do you do?

a. Don’t do anythingb. Call her in 14 daysc. Send her home a formal follow up letter

Page 20: Feb 2012 AMCQ Recall

199.Old man with a terminal illness agrees to receive supportive treatment. His son, who is not living with him and is not his attorney of medical advice, wants Sx for him.

a. Ask the son to go to medical legal boardb. Follow patient’s wishesc. Discuss with the son pros and cons of the operation and convince him

200.Pic of Erythema Multiforme201.Pic of drug rash on legs.

202.Pregnant lady with hyperthyroid findings. What to prescribe to her?a. PTU (propylthiouracil)b. Carbimazole

203.Pt agitated and breathless immediately after operation. What to do?a. Remove superficial suturesb. Explore wound in the wardc. Call the surgeond. Do nothing

204.Boy fell from the tree; he is having a clear fluid discharge from the nose. CT is needed. ‘Big’ hospital is 4 hours away by road. What will you do?

a. Send him to nearest hospital by ambulanceb. Air ambulance the Ptc. Tell the Pt you can’t do CT

205.Jehovah witness Pt needs blood transfusion for Ph incompatibility. Relatives say ‘no’. What do you do?

a. Transfuse the Ptb. Do not transfuse the Ptc. Discuss the situation with relativesd. Ask the Pt to sign ‘ No transfusion form’

206.Pt on warfarin, INR 4 (2.5-3.5) came to hospital with pain in left side of the chest. There is tenderness in LIF, no gardening. CT: rectus sheath hematoma. Dx?

Pt after trauma becomes agitated at hospital and wants to run around. What will you do?a. Pull sides of his bed upb. Restrainc. Do one to one nursing

207.Pt comes with peripheral weakness and has difficulty seen at night. What would you give her initially?

a. Folic acidb. Iron supplementsc. Vit B12

208.WPW + VT. Mx?a. Sotalolb. Amiodaronec. Adenosine

209.Pic of venous ulcer.a. Arterial ulcerb. Venous ulcer

Page 21: Feb 2012 AMCQ Recall

210.A man with a low visual acuity, which becomes worse in sunlight. Pic is given.a. IOL (intraocular lenses)b. Phacoemulsificationc. Pilocarpind. Keratoplastye. Acytozolamide f. Trabeculectomy

211.Pic of apical TB in lungs. A rural nurse comes with hemaphtysis. Dx?a. Bronchitisb. TBc. Bronchogenic Ca

212.A girl becomes dizzy few times while at the market. Becomes ok in 2-3 minutes. Dx?a. Migraine b. Arrhythmia

213.A plumber comes with cough. He is a smoker. CXR given: changes in pleura. What is definitive Ix?

a. Bronchoscopy and Bronchoalveolar Lavageb. Pleural biopsyc. CT chest

214.6y.o. girl comes with pubic hair and breasts in Tanner stage 1. What will you do?a. Reassure and see her in 6 monthb. Do FSH and LH

215.A 6y.o. girl is very scared vigilant anxious; low grades at school. Her teacher is worried. Next step?

a. Talk to mother aloneb. Open discussion with both parentsc. Take more information from the teacher

216.Bone pain. Very high ALP/ALKP; no Ca ++ given. Next Ix?a. Ca ++b. Electrophoresis

217.Mallet finger. Mechanism of injury.218.‘Big stem’: Pt with Hx of gastrectomy; all lab tests are given: low Hb + high MCV. Dx?

a. Fe deficiency b. Vit B12 deficiency

219.Woman with vaginal discharge, no fever, cervical excitation.a. PID

220.Pic: Tinea capita

221.Pseudobulbar palsy which feature is absent?a. Absent Gag reflex

222.Nephritic syndrome becomes steroid resistant. Dx?a. IgA nephropathyb. Post-streptococcal GNc. Focal segmental GS

Page 22: Feb 2012 AMCQ Recall

223.Q8 from Peads Nov’11:224.A 2 week old child with temp of 40'c came to hospital w hx of seizure + looking ill. Mother very concerned as child not feeding well. What is the diagnosis?

a) febrile convulsion b) septicaemia c) encephalitis d)epilepsy

225.Pt with weakness in one hand + headache. Dx?a. Atypical migraineb. Migraine

226.32y.o. pt with rash on buttocks + fever + …a. HSPb. ITPc. Hemolytic uremic syndrome

227.10 month old child has been weaning + started cow milk and solids. On presentation pale and unhappy + lost 2 kg. Dx?

a. Celiac diseaseb. Cow-milk allergyc. Cystic fibrosis

228.10 month old unhappy baby. Wt - 10th percentile, Ht - 25percentile.a. Celiac diseaseb. Cow-milk allergyc. Cystic fibrosis

229.Child, normal delivery, presents to GP for a checkup. On exam: pansystolic murmur. Dx?a. Tetralogy of Fallot b. ASDc. VSDd. TGA

230.4y.o. aboriginal child speaks 5 words only + nose and ear discharge. What do you do next?

a. Nasal swabb. Ear swabc. Hearing testd. Tympanometry

231.Which one of these disorders is affected by/ related to culture?a. Schizophreniab. Anorexia nervosac. Agoraphobia

232.Lewy body dementia: short stem (4-5 sentences) mentioning visual hallucinations.Elderly Pt in nursing home is abusing nurses while they shower him, masturbates a front of the nuking station. What do you do?a. Frontal lobe dementiab. Alzheimer’s diseasec. Lewy body dementia

233.Old lady in a nursing home masturbates privately. What do you do?a. Complain to nurse’s supervisor

Page 23: Feb 2012 AMCQ Recall

b. Leave her alone 234.Child with 5-9% dehydration.

a. NG dehydration

235.HIV , 2 homosexuals. Best method for protection.a. Condom

236.Fundoscopy: 76y.o. Pt with CRAO

237.For H. pillory medications: a/b with PPIs will help to:a. Increase the relapse timeb. PPIs help a/b to penetrate the ulcer

238.2y.o. child with bloody diarrhea, n & v, swellings, petechiae, low urine output. Now lethargic. Dx?

a. Hemolytic uremic syndromeb. Salmonella diarrhea c. Bacterial septicemia

237.Several faintings in old Pt. P/E: HR 44; BP 180/110; ejaculating murmur. What is the cause for fainting?

a. ASb. CHB

238.Pt comes with h/o of treatment of osteoarthritis with Celecoxib, develops mouth ulcers. Mx?

a. Cease Celecoxibb. Give folic acid

Viral meningitis: CFS findings?% of burns in a child (picture is given).Ischemic abdo pain. Mx?

Pictures from ‘Anthology’:HCCRectus sheath hematomaRA XRBurns (p.112)BBC (pinna of ear)Colles #Ulnar distal #Spine # (osteoporosis)Pictures from ‘Blue Book’:Acanthosis nigricansBulemia nervosa

239.Female Pt right hip. How to use stick?a. Hold in left hand and put your right leg forward

240.Pt coming with pain on defecation, pruritis and hemorrhoids seen only on anal wedge. Mx?

a. Increase fiber in dietb. Rubber band ligationc. Sclera therapy

Page 24: Feb 2012 AMCQ Recall

241.Pt with dementia + critical illness. Conflict between wife’s and son’s wishes.a. Go with wifeb. Go with sonc. Pros and contras of the condition

242.A girl comes for an OCP prescription, wants to have safe sex. Periods are just started. She tells mother and sister has menorrhagia problems. What do you do?

a. Prescribe OCPb. Screen her for coagulopathyc. Tell her to use OCP and condom

243.Tocolytics contraindications:a. Antepartum hemorrhage b. Previous C-section

244.Pt doesn’t want resuscitation. What do you do?a. Write “DNR” sign on his badb. Check his mental statusc. Ask him to sign the form

245.Pt diagnosed with colon Ca Duke C stage. After Sx what do you do?a. Pelvic examinationb. ChemoTx c. Colectomyd. 5-FU (5-fluorouracil)

246.Pt underwent appendectomy. Histology showed carcinoid tumour of 2cm. What do you next?

a. ChemoTx b. RadioTxc. Rt hemicolectomyd. Leave it as it is

247.ECG od SVT. Rx?a. Amiodaroneb. carotid massagec. adenosine

248.Pt with VFib, ECG given. Mx?a. Atropine b. Adrenalinec. Lignocaine

249.Pt with sore thoat treated with amoxicillin. After 4 days presented with rash. Dx?a. Allergic reactionb. SLE

250.Female Pt presents with peripheral vascular disease. H/o DM and HT + was a smoker, till 3 years ago she developed DVT syndrome and quitted. Dx?

a. Berger’s diseaseb. Raynaud’s phenomenac. SLE

Page 25: Feb 2012 AMCQ Recall

251.Chronic PID Mx?IV drug user arrives to hospital with high fever. Dx?a. Rheumatic feverb. Endocarditisc. Pneumonia

252.The nurse in a nursing home advises you what to prescribe to the Pt. What do you do?a. Talk to the nurse during the breakb. Talk to the nurse supervisorc. Write a formal complaint to the nursing home administration

253.Coal miner, working since 40 years, no smoking Hx. CXR : pleural effusion. Dx?Infected Bartholin cyst? Mx254.Pt with Paget’s disease. What is it associated with?

255.Infected Bartholin cyst. What is true?a. Is asymptomaticb. Treated by a/bc. Caused by gonococcus

256.Old man with Hx of confusion and forgetfulness. He is driving at night. What is your advise?

a. Stop driving

257AFlatter: full ECG – Dx?

258.You are running investigation/survey: how many people in the area are having a particular disease (DM or Hypertension). WOF will be the best method?

a. Case control studyb. Randomized study

259.Husband is diagnosed with hereditary haemochromatosis. Wife is healthy. They have two children: 9y.o. and 18y.o. Whom are you going to screen?

a. 9y.o.b. 18y.o.c. Both

260.How many children will be affected by Cystic fibrosis, if one of the parents is a career?a. 0b. 0.5c. 0.25

261.Pt complains of developing digital ulcers when exposed cold in winter. Tx?a. Hydroxychloroquineb. Oral prednisolonec. IM prednisolone

262.What are the risk factors of endometrial Ca?a. Obesityb. Smokingc. OCPd. Multiparity

Page 26: Feb 2012 AMCQ Recall

263.Pt agitated and confused, brought in to ED. What will you give?a. Haloperidolb. Olanzepine c. Diazepamd. Fluoxetine

264.Old man living alone, wife died few years ago. Doesn’t want to live. WOF would be of more concern?

a. He is looking for suicidal methods on internetb. His any previous suicide attemtsc. He is living alone

265.Pic of Staphylococcal pneumonia.ECG of VT. Mx?a. Carotid massageb. Amiodaronec. Verapamil

266.Mother brought the child with some problem. Doctor ordered CT. mother couldn’t bring her child for the appointment. What do you do?

a. Don’t do anythingb. Call her in 14 daysc. Send her home a formal follow up letter

267.Old man with a terminal illness agrees to receive supportive treatment. His son, who is not living with him and is not his attorney of medical advice, wants Sx for him.

a. Ask the son to go to medical legal boardb. Follow patient’s wishesc. Discuss with the son pros and cons of the operation and convince him

Pic of Erythema MultiformePic of drug rash on legs.

278.Pregnant lady with hyperthyroid findings. What to prescribe to her?a. PTU (propylthiouracil)b. Carbimazole

269.Pt agitated and breathless immediately after operation. What to do?a. Remove superficial suturesb. Explore wound in the wardc. Call the surgeond. Do nothing

270.Boy fell from the tree; he is having a clear fluid discharge from the nose. CT is needed. ‘Big’ hospital is 4 hours away by road. What will you do?

a. Send him to nearest hospital on ambulanceb. Air ambulance the Ptc. Tell the Pt you can’t do CT

Page 27: Feb 2012 AMCQ Recall

271.Jehovah witness Pt needs blood transfusion for Ph incompatibility. Relatives say ‘no’. What do you do?

a. Transfuse the Ptb. Do not transfuse the Ptc. Discuss the situation with relativesd. Ask the Pt to sign ‘ No transfusion form’

272.Pt on warfarin, INR 4 (2.5-3.5) came to hospital with pain in left side of the chest. There is tenderness in LIF, no gardening. CT: rectus sheath hematoma. Dx?

273.Pt after trauma becomes agitated at hospital and wants to run around. What will you do?a. Pull sides of his bed upb. Restrainc. Do one to one nursing

274.Pt comes with peripheral weakness and has difficulty seen at night. What would you give her initially?

a. Folic acidb. Iron supplementsc. Vit B12

275.WPW + VT. Mx?a. Sotalolb. Amiodaronec. Adenosine

276.Pic of venous ulcer.a. Arterial ulcerb. Venous ulcer

277.A man with a low visual acuity, which becomes worse in sunlight. Pic is given.a. IOL (intraocular lenses)b. Phacoemulsificationc. Pilocarpind. Keratoplastye. Acytozolamide f. Trabeculectomy

278.Pic of apical TB in lungs. Rural nurse comes with hemaphtysis. Dx?a. Bronchitisb. TBc. Bronchogenic Ca

279.A girl becomes dizzy few times while at the market. Becomes ok in 2-3 minutes. Dx?a. Migraine b. Arrhythmia

280.A plumber comes with cough. He is a smoker. CXR given: changes in pleura. What is definitive Ix?

a. Bronchoscopy and Bronchoalveolar Lavageb. Pleural biopsyc. CT chest

Page 28: Feb 2012 AMCQ Recall

281.6y.o. girl comes with pubic hair and breasts in Tanner stage 1. What will you do?a. Reassure and see her in 6 monthb. Do FSH and LH

282.A 6y.o. girl is very scared vigilant anxious; low grades at school. Her teacher is worried. Next step?

a. Talk to mother aloneb. Open discussion with both parentsc. Take more information from the teacher

283.Bone pain. Very high ALP/ALKP; no Ca ++ given. Next Ix?a. Ca ++b. Electrophoresis

284.Big stem’: Pt with Hx of gastrectomy; all lab tests are given: low Hb + high MCV. Dx?a. Fe deficiency b. Vit B12 deficiency

285.Woman with vaginal discharge, no fever, cervical excitation.a. PID

Pic Tinea capita

286.Nephritic syndrome becomes steroid resistant. Dx?a. IgA nephropathyb. Post-streptococcal GNc. Focal segmental GS

Q8 from Peads Nov’11:

287.A 2 week old child with temp of 40'c came to hospital w hx of seizure + looking ill. Mother very concerned as child not feeding well. What is the diagnosis?

a) febrile convulsion b) septicaemia c) encephalitis d)epilepsy

288.Pt with weakness in one hand + headache. Dx?a. Atypical migraineb. Migraine

289.10 month old child has been weaning + started cow milk and solids. On presentation pale and unhappy + lost 2 kg. Dx?

a. Celiac diseaseb. Cow-milk allergyc. Cystic fibrosis

290. 10 month old unhappy baby. Wt - 10th percentile, Ht - 25percentile.a. Celiac diseaseb. Cow-milk allergyc. Cystic fibrosis

Page 29: Feb 2012 AMCQ Recall

291.Child, normal delivery, presents to GP for a check up. On exam: pansistolic murmur. Dx?

a. Tetralogy of Fallot b. ASDc. VSDd. TGA

292.4y.o. aboriginal child speaks 5 words only + nose and ear discharge. What do you do next?

a. Nasal swabb. Ear swabc. Hearing testd. Tympanometry

Fundoscopy: 76y.o. Pt with CRAO

293.For H. pillory medications: a/b with PPIs will help to:a. Increase the replse timeb. PPIs help a/b to penetrate the ulcer

294.2y.o. child with bloody diarrhea, n & v, swellings, petechiae, low urine output. Now lethargic. Dx?

a. Hemolytic uremic syndromeb. Salmonella diarrhea c. Bacterial septicemia

295.Several fainting in old Pt. P/E: HR 44; BP 180/110; ejaculating murmur. What is the cause for fainting?

a. ASb. CHB

296.Pt comes with h/o of treatment of osteoarthritis with Celecoxib, develops mouth ulcers. Mx?

a. Cease b. Celecoxib

297.What is true about HIV.a. Antibodies appears after 9 yearsb. Risk transmission to foetus is 50%c. Ratio CD4 to CD8 is more than 5

300. A middle aged lady w hx of pain over distal fingers - more on palpation with no obvious deformity for 20 years + Anaemia + CXR shows enlarged heart .what is the diagnosis?

a) scleroderma b)SLE

301.A man post gastrectomy 3 years ago c/o lethargy. On examination, pallor+ .blood ix: hb 9.0 mcv:110 all other normal range .Wht is the diagnosis?

A) iron def Anemia b) vit B def Anemia c) chr Anemia

Page 30: Feb 2012 AMCQ Recall

302.A ECG w WPW + vent tachycardia .Wht us the mx?a) adenosine b)amiodarone c)b blocker d)verapamil

303.A women w hx of passing small quantities of urine after bouts of cough each time was sent fr investigation.Wht is the best investigation to come to a diagnosis?

a)ct abd n pelvis b)laproscopyc)cystoscopy

304.Patient complains of jaundice with dark urine and pale stools. Pain in right upper quadrant. There is a mass which moves with respiration. Most likely diagnosis?

a) Choledocolithiasisb) Carcinoma of ampulla of pancreasc) Mucocele

305.Patient with mass above the umbilicus . She cannot reduce the swelling anymore and it is painful. She is on warfarin and posted for surgery. INR is 2.9. What will you do ?

a) Stop warfarin, start heparin and do surgeryb) Stop warfarin, give FFP and do surgeryc) Give vit k and delay surgery for 7 days d) Delay surgery for 3 months

306.which feature would be absent in pseudarbulbar palsy?a)Dysarthriab)Dysphagiac)Absent Gag Reflex

307. Year old man, was scrathed by his cat, and 7 days after there was a pustules from humerus to thumb and oedema of arm at the site of the scrath . What could be the diagnosis?

a)Cellulitisb)cat scrath feverc.Herpes d.Strep

308.Inferior Wall MI , Of a 55 year old man, with history of Hyperension. Patient is in a Metropolitan Hospital. Patient has been given Oxygen, Nitrates , Morphine and Aspirin. Patient came to the hospital after 4 hours of symptoms. what would be the most appropriate next step in the management of this patient.

a) Check Troponin right now and 8 hours later b)Coronary Angiography

309.Patient who was on digoxin for atrial fibrillation was given hydrochlothiazides for Hypertension. About 3 days later patient complains of nausea, vomiting, diarrhoea and patient complains of palpitations. What would be the most appropriate next step in management?

a) stop hydrochlothiazides and give pottassium supplements. b) stop digozin and give pottassium supplements

Page 31: Feb 2012 AMCQ Recall

310.A 30 year old woman comes to your clinic with complaints of a seizure like activity which occurred last night. She said her seizures were well controlled the past 3 years with the antiepileptic medication and again they reappeared since recently. How long should she wait to drive?

a) 6 months seizure free period b) 2 years of seizure free periodc) she can drive straight away

311.A 30 year old lady comes with a bruise on the left cheek and a cut on the lip. If you check which system would you be able to come to a diagnosis?

a) CVSb) Respiratory Systemc) Central Nervous Systemd) Musculo skeletal system

312.Chest Xray showing a cavitary lesion at the apical part of the right side lung in a tourist coming from sudan. he comes with fever and night sweats. sputum doesnt show any acid fast bacilli. what would be the most appropriate step in the management?

a) Isoniazid prophylaxisb) Isoniazid, Rifampacin, Pyrazinamide and ethanbutamol c) Amoxicillin

313.A HIV pt came to your clinic along with partner and asking for the best prophylactic measures they should take…

a. Use condom and safe sexb. both should use anti HIV drugs

314.A driver from interstate has moved to your locality recently and is stable on lithium for the past 15 yrs.How do you monitor him?

a. check lithium levels every 3 monthsb. check lithium levels every 6 monthsc. check lithium levels every monthd. full blood count

315.Picture of man w normal eyes.History of watery discharge.what is the management?a. Chloramphenicol ointmentb. cold compressc. Acetazolamided. Acyclovir drops

316.85yr old patient presented with right thigh and leg pain after fall.Investigations are as follows—Ca 3.5,Urea 10.Most common cause

a. Osteoporosisb.Vit D toxicityc. Primary hyperthyroidism

317.A middle aged women with calf pain from few days came to hospital.her investigations showed Deep vein thrombosis.What is the management?

a. IP management with compression with compression socking and LMWHb. IP management with elevation and LMWHc. OP management with compression and LMWHd. OP management with leg elevation and LMWH

Page 32: Feb 2012 AMCQ Recall

318.Middle aged man with history of cough.Cxay showed---lower lobe consolidation.What is the management?

a. oral amoxicillin and clavulanateb. oral azithromycinc.IV ampicillind. Oral flucloxacillin

319.Sixth nerve palsy picture given.Diagnosis?320.Amitriptyllin intoxication.What is the management?

a. IV fluidsb. Intubation and Sodium bicarbonatec. Gastric lavaged. Activated charcoal

321.COPD patient received 6L oxygen and get worse. What is the next step?a. Immediate Cxrayb. Reduce oxygenc.Increase oxygend CPAP

322.A patient on following medications---thiazide 25 mg, B-blocker presented with polyuria, polyuria. His blood sugar 17 mmol/l. What is next management?

a. Stop thiazide and reassessb. Start on oral hypoglycemicc. insulin treatment

323.patient with severe chest pain with half an hour history. Ecg was done which was normal, cardiac markers were normal too. what is most appropriate step in management of this patient?

a.send him homeb.give GTNc.do pcid.stress test

324.Young slim marathon runner with a hx of 2 hour sudden onset of left sided chest pain and shortness of breath on mild exertion. dx-

a.MIb.spontaneous pnemothoraxc.cardiomyopathy

325.A man came from Sudan for a visit; Came to your practice complaining of dry cough, weight loss, night sweating. chest X-ray given that shows right lung apical cavitation. WOF is the most appropriate management:

-Isoniazid-Quadri-TB therapy-Chest CT

326.35 years old women with h/o headache and bi-lateral homonymous hemianopia. She recovered from headache but the h/h still persist after 6 weeks. She came to your practice asking when she can drive her car :

-after 3 months -after 6 months -never drive again

Page 33: Feb 2012 AMCQ Recall

-after 1 year

327.Young lady presents to GP early morning with history of 8 glasses of spirits last night, with history of nausea since last night. Today morning nausea with vomiting. P/E normal. Next step in Mx?

-Review in 4 hours-Review later in the day-explain this is due to excess alcohol and no need to worry-prescribe anti-emetics and review if symptoms recur

328.Pt with Osteoporosis and oesophagitis. What is the treatment? A. alendronateB. zoledronic acidc) Increse milk intaked) risedronatee) Give Calcium supplement.

329.What is best treatment child with petit mal.- Carbomazepine- Na valproate- Barbiturates- Diazepame- No option with ethosuccinate

330.A man on treatment with warfarin for DVT, 2 days ago roxithromycin was added and now INR-9.Pt has mild rectal bleeding, what would be the next initial management?

a) Change to another antiboitic.b) Give FFPc) Stop warfarind) Cryptococcus

331.IN a patient with GBS which is ah appropriate way to monitor his oxygenation?a) forced vital capacity in 1secb) forced expiratory capacity in 1 secc) blood gasesd) pulseoxymetere) Oxygen saturation

332.A young lady had headache after she wake up this morning, she vomited once 10am and abdominal pain 1200. She had alcohol drinking last night. Your examination is normal. What is the next step.

A) tell her is normal, and send her home.B) Tell her it is alcohol related. And send her homeC) Tell her the diagnosis is uncertain, review her after 4 hours.

333.WPW syndrome with ventricle ectopic. What is your next step in management.a) Adenosineb) Verapamilc) Amioderon

334.A pt came with headache and tightness on the scalp. All vital sings and physical, neuro examination are normal. What is the next step management.

A) temporal artery biopsy.B) prednisoloneC) CT

Page 34: Feb 2012 AMCQ Recall

335.Patient who was on digoxin for atrial fibrillation was given hydrochlothiazides for Hypertension. About 3 days later patient complains of nausea, vomitting, diarrhea and patient complains of palpitations. What would be the most appropriate next step in management?

a) stop hydrochlothiazides and give pottassium supplements. b) stop digoxin and give pottassium supplements

336.A 30 year old lady comes with a bruise on the left cheek and a cut on the lip. If you check which system would you be able to come to a diagnosis?

a) CVSb) Respiratory Systemc) Central Nervous Systemd) Musculo skeletal system

337.Patient complains of jaundice with dark urine and pale stools. Pain in right upper quadrant. There is a mass which moves with respiration. Most likely diagnosis?

a) Choledocolithiasisb) Carcinoma of ampulla of pancreasc) Mucocele

338.Patient with mass above the umbilicus. She cannot reduce the swelling anymore and it is painful. She is on warfarin and posted for surgery. INR is 2.9. What will you do ?

a) Stop warfarin, start heparin and do surgeryb) Stop warfarin, give FFP and do surgeryc) Give vit k and delay surgery for 7 days d) Delay surgery for 3 months

339.A lady on warfarin developed Rt iliac fossa pain.CT scan given. Dxa)rectus sheath haematomab)appendicitisc)mesentric eschemia

340.A picture of Keratoacanthoma. [ Lifted from the AMC Anthology book- exactly the same in the eye] What is the Diagnosis?

A. Pyoderma granulosaB. SCCC. BCCD. Nodular MelanomaE. Keratoacanthoma

341.Same picture as above. What is the next step in management. A. Wide local excision of at least 4mm marginB. Local incisionC. Local excision with sentinel node biopsyD. Refer to plastic surgeryE. Review after 6 weeks

342.Ct scan showing Intercerebral Haemorrhage , patient was on warfarin and his INR at the moment is 4.9. What would be the most appropriate next step

a) give vitamin K b) GIVE protamine sulphate c) Give FFP

Page 35: Feb 2012 AMCQ Recall

343.A 60 year old man with a long history of diabetes, comes to the hospital with ulcer in each dorsum of the toe, in both the legs. There is foul smelling discharge from the right side ulcer. What would be the most appropriate next step in management?

a) Amputate the toes.b) Do Doppler Ultrasoundc) Give antibiotics (Amoxicillin and Clavulonic Acid)

344.A 50 year old man comes with a swelling in the groin region. its reducible on lying down and patient says its slightly tender. patient was put on aspirin and clopidogrel 3 months back for atrial fibrillation. what would be the most appropriate next step in the management?

a) five vitamin k and send the patient immediately for surgeryb) give ffp and surgeryc) stop aspirin and clopidogrel for 1 week and then proceed with the surgery.d) no need of any medications , just refer the patient immediately for surgery.e) nothing needs to be done.

345. Patient comes to you with complaints of swelling under the jaw, and its more prominent after food. Bimanual Palpation was done and what would be the most appropriate next step?

a) Intraoral Xray of mandibulab) CT scanc) Sialogramd) OPGE) US

346. A 70 year old woman of Ca colon, comes with lethargy , feeling tired ,and altered bowel habits. which feature would appear 1st in cancer of the caecum?

a) palpable massb) altered bowl habbitsc) Anaemia

347. 35 year old man comes saying, his father was diagnosed with prostate cancer at 85years age, and neighbour was diagnosed with the same condition recently. hes worried he might get prostate cancer. On digital Rectal Examination, his prostate was smooth ,soft and median sulcus was felt.He has no symptoms suggestive of prostate cancer. What would you advice to this patient?

a) Reassuaranceb) PSAc) DRE annually

348.Picture of venous ulcer, which one of the following are the risk factors?a. smokingb. Previous history of DVTc. Coronary aretery disease

349.Mallet finger –Mechanism of injurya. flexion injuryb. hyperflexion injuryc. hyperextention injury

350.History of foreign body in the eye. What is the initial management?a. remove the foreign body with the needleb. Local anaesthetic drops and irrigate with fluidsc. antibiotic dropsd. miotic drops

Page 36: Feb 2012 AMCQ Recall

351.Patient with earache, swelling in the cheek. On examination—ear—N,What is the next intial investigation?

a. OPGb. Head CTc. sialogramd.CTe. X-ray mandibulae

352.35 yr old male who noticed his right testes was bit swollen and larger than left one. on exam everything looked ok. Dx-

a. epididymo orchitisb. tortion of testesc. Hydrocele

353.24 years old with severe knee pain. On examination you found effusion around the knee.. pain on knee movements. dx

a.anterior cruciate ligament injury’b.post. cruciate ligament injuryc.medial meniscus injuryd.lateral meniscus injury

354. 5 yo child had lower limb injury until the fat layer. He has 2 boots his anti-tetanus vaccination. What is the most appropriate management after wound debridement:

-toxin and Ig-aDTP and Ig-Ig-reassure

355.An executive undergone major surgery. After 72 hours Post-op, agitated, disturbed. Says strangers entering his room, while you know, nobody entered his room. Whats the cause?

-S/E of medication-Alcohol withdrawal-Psychotic exacerbation-Chest infection

356.Elderly patient with history of anaemia and respectively symptoms of it, no other historyor symptoms. What is the next step investigation?- gastroscopy and colonoscopy-sigmoidoscopy-colonoscopy

357.Middle age man after binge drinking in a party suddenly presents with pain abdomen, vomittings. P/E P-110, BP 90/40, tenderness and guarding Lt hypochondriac region, reduced breath sounds & dull percussion at Lt lower chest . What is the cause?

-oesophageal tear-ruptured peptic ulcer-spontaneous pneumothorax-Hiatus hernia

358.A 18 years old male patient presented with painless swelling of scrotum which he realized after sport. But no injury was recorded. What is the diagnosis?

a) Hydrocele. b) teratomac) Seminomad) Torsion of testis

Page 37: Feb 2012 AMCQ Recall

359.A teenage boy came with pain in his right knee after knee injury in the sport game. His knee is red, swollen, hot, painful. What is the diagnose tool?

a) CTb) MRIc) Aspiration of the kneee) X-ray

360.A female patient had trauma. She got multiple rib fractures on her left side from. She is conscious and well. Her physical examination is dull on percussion on the left side, and decrease the breath sound on the same side. What is your next step of Mx?

a) Insert a cheat tube and drain.b) Intubate and ventilate

361.Pt w 15% pnemothorax. Mx.a. Reassurance and observeb. Drainage under seal waterc. Oxygend. Intubatione. aspiration

362.A 2 week old child with temp of 40'c came to hospital w hx of seizure + looking ill. Mother very concerned as child not feeding well. What is the diagnosis?

a) febrile convulsion b) septicaemia c) encephalitis d)epilepsy

363. A 18 month old girl who was started on solid foods and cow's milk since 10 months ago is seen not well and had lost considerable weight at presentation. How will a confirmatory diagnosis be expediently achieved?

A. Cow's milk cessationB. Doudenal biopsy

364. A child is brought with complaints of rectal bleeding, history of opening bowels once 4 to 5 days.What is the treatment?

a. Laxative and diet rich in fibreb. Increase fluidsc.proctoscopyd. colonoscopy

365. Child with history of URTI 3 weeks ago,now presents with ptechiae,bruises.Platelets decreased. All other on investigations Normal. What is the management?

a. Strict bed restb.gamma globulin c. steroidsd.antibiotics

366. A 14 years old boy h/o nephrotic syndrome not responding well to cortico-steroid therapy, urine protein-++,hypertension, hematuria with red casts. WOF can be the cause:

A .Ig A nephropathyB .PSGNC .focal glomerulosclerosisOBS & GYN

Page 38: Feb 2012 AMCQ Recall

367. Wht is true regarding infected bartholin cyst?a)maybe caused by gonococcal infection b) treat w antibiotics c) surgical removald) caused by Stahpe) caused by Strep

368. A women w hx of passing small quantities of urine after bouts of cough each time, was sent for investigation. Wht is the best investigation to come to a diagnosis?

a)ct abd n pelvisb)USG pelvisc)Cystoscopy

369. A lady on sodium valproate w hx of well controlled seizure during pregnancy and BP 165/85.post delivery day 2-developed generalised seizure .protein +3 in urine. what is the diagnosis?

a) generalised seizure b)eclampsia c)hypoglycaemia

370. A long stem wanted to know …what would indicate the delivery is most likely to occur before 16wks of gestations.

a. Fetal fibronectine test on cervical secretion is positiveb) Vaginal dischargec) Absent Fetal heart soundsd) US of cervix length

371. A lady 37yrs old comes for prescription for OCPs. She is known smoker, no other systemic illness. What advice would give this lady about smoking?

a. Give her bupropion tablets.b. give her nicotine chewing gums or patchc. advise her about the benefits of quitting smokingd. refer her to de-addiction centre

372. A women is advised by her GP to take OCP for next 5yrs.Which of the following is applicable considering that she no risk factors?

a. Increased risk of breast Cancer by 25%b. Increased risk of stroke by 10%c. Increased risk of osteoporosis by 5%d. Increased risk of heart disease <1%

373. A pregnant woman with vesicles in the vagina, have difficulty in micturition. On lab inv—Herpes infection. What is the management?

a. oral acyclovirb. IV acyclovirc. Interferond. Oral fancyclovir

374. DX for FT ectopic prega.US of visral organb. Bhcgc. Laproscopyd. CT abdo

Page 39: Feb 2012 AMCQ Recall

375. Preg lady …10 wks … uterus size 14 wk… best Dx method.a. B hcgb. USGc. CT

376. pregnant lady detected with HIV , viral load undetected.CD4 count was given otherwise well , on no medication .which is inappropriate advice?

a. highest risk of transmission is likely with low CD4 count.b. high maternal plasma HIV RNA increases transmission risk.c. transmission risk to foetus is high during antenatal period .

377. 21 year old at 26 week gestation with lower abdominal pain n tenderness (Rt iliac fossa). Appendectomia was done 10 y. Ago dx-

a. pyelonephritisb. appendicitisc. degeneration of unknown uterine fibroidd. ectopic pregnancye. uknown cause

378. 52 years old women with a BMI-33 , h/o amenorrhea since 12 months what do you expect to find in her lab analysis:

a. high estrogen and low FSHb. very low estrogen and very high FSHc. slightly low estrogen and slightly high FSH

379. Young women came to ER for severe lower abdominal pain, low BP-90/50, pulse-110,PV bleeding,Hb-90,last menstrual period was 8 weeks ago. WOF will leads you to suspect ectopic pregnancy:

a)low BPb)lower abdominal guardingc) PV bleedingd) low Hb

380.Management of abruptio placenta with death foetus:- cesarean-amniotomy-expectant ( Observe )

381. 2 weeks postpartum lady present with psychotic episodes. She is breast feeding. Best treatment: -

-Olanzapine- Lithium-diazepam

382. Young lady with DVT comes to GP for contraception. Apart from condom advice, what else you will add?

-spermicidal gel-continuous OCP-Estrogen patch-Progesteron only pill

Page 40: Feb 2012 AMCQ Recall

383. pic of pilonedal sinsus in blu book page 207 asked diagnosis exect same pic

384.19 year old boy complains of having difficulties in playing sports at school. His height is at 97th percentile for his age and weight is at 3rd percentile. What is your next step in management -

a) Karyotyping b) bone scanc) growth hormone testd) CT Scane) Cardiac ultrasound

385.a pt who worked in a gold mine for 20 years and is smoker and history of working in petrol station for 20 years now presents with hematemasis and cough. Xray was given, how would diagnosis him

A) Bronchoscope B) USGC) Ct scan D) Needle aspiration

386.A Ct scan of a 80 years old man was given ( massive bleed with displaced ventricles, is admitted to the ICU, pts wife want a calm ending for him while the Son want to do want ever it takes to save his father. Who would decide on the treatment?

a) Follow the wife wishes b) Follow the son wishes c) Decide by your own using the doctor powerd) Arrange family meetinge) Guardian ship court

387.A middle aged lady w hx of pain over distal fingers - more on palpation with no obvious deformity for 20 years + Anaemia + CXR shows enlarged heart .what is the diagnosis?

a) scleroderma b)SLE 22) A picture of psoriasis of the leg with extensive lesions. What is the treatment of choice?a)UVB light b) calcitriol

388. A pic of SCC at lower lip, what is most common cause?a) Smokingb) Sun exposurec) Metsd) Unknown cause

389.A critically ill pt who mad advance directives when he was in good health now no tubesNow he has a massive stroke and critically ill. His family also agreed with his decision previously made. How will you decide on his rx nowa) Intibuate and ventilateb) IV line and feedc) Cvp and fluidd) Oral small sips of watere) Send him for terminal care

Page 41: Feb 2012 AMCQ Recall

390.A pt had an accident his pulse is 120 BP is 120/80 laying and 90/70 standing what is the percentage of blood loss

a) 10 %b) 20 %c) 25%d) 35%e) 40%

391. a 18 month old has head circumference 25th percentile at birth 50th percentile at 6 moth 75th percentile at 1 year and 97th percentile at presentation what is the best way to diagnose.

a) Cranial USGb) MRI c) No need to investigate

392. ECG of SVT from blue book asked diagnosed393. old home pt with off and on constipation and Xray of the sigmoid volvulus on blu book page 458 was given and asked for diagnosis

a) sigmoid volvulusb) lage bowl obstructionc) small bowl obstructiond) adhission

394. ecg of WPW asked for rxa) Carotid messageb) amidrionc) digoxind) adenosine

395.SVT ECG from blue book and dx asked.

396. ECG of an old lady with SVT and then sudden a systole after carotid message . asked for tx

a) amioderonb) adenosinec) shift her to ICUd) atropine i/ve) do not do carotid message again.

397.venous ulcer asked about the most common associationa) smokingb) DVTc) long bone fracture

398. a pic of ulcer o the medial meallolus what the diagnosisa) arterial ulcer b) venous ulcrc) diabetic ulcer

399.Q about the CTG interpretation what is more likely associated with foetal hypoxia. Ca not remember the option

Page 42: Feb 2012 AMCQ Recall

400.Patient complains of jaundice with dark urine and pale stools. Pain in right upper quadrant. There is a mass which moves with respiration. Most likely diagnosis?

a) Choledocolithiasisb) Carcinoma of ampulla of pancreasc) Mucocele

401.an old pt on who just had drug elucidating prosthetic valvein place is on clopidrogril came in to preoperative clinic for evaluation. He has a non complicated vericocele. Wht will you advice him regarding the operation

a) reacess the nedd for durgeryb) stop clopidrogil and shift to heparin one week before surgeryc) give FFP before surgeryd) stop clopidrogril few days before surgery e) giv vit K

402. acute limb ischemia .what sign would require for urgent surgery?a) paresthesia b)paralysis c)rubor

403. A lady of a certain age had Subtotal Thyroidectomy several hours ago. She was noted to have stertous respiration in the ward. There is some swelling and redness on the post-op site. What is the next step in the management?

A. Perform bedside Ultrasound B. Remove the deep muscle sutures.C. Call the Surgeon and arrange intra-op exploration of the wound siteD. Request CXR urgently E. Remove the skin staples

404.A 60 year old man with a long history of diabetes, comes to the hospital with ulcer in each dorsum of the toe, in both the legs. There is foul smelling discharge from the right side ulcer. What would be the most appropriate next step in management?

a) Amputate the toes.b) Do Doppler Ultrasoundc) Give antibiotics (Amoxicillin and Clavulonic Acid)

405.A 30 year old man with multiple rib fractures following a motor vehicle accident. Having paradoxical breathing. what would be the most appropriate next step in management?

a) strap the chest wellb) CPAP with Ventilationc)no option for intubation and ventilation

406. pic of the old lady with a red swollen check like cellulites what is the mx.a) Incision and drainageb) Antibioticc) OPG

407.A teenage boy came with pain in his right knee after knee injury in the sport game. His knee is red, swollen, hot, painful. What is the diagnose tool?

a) CTb) MRIc) Aspiration of the kneed) antibiotic

Page 43: Feb 2012 AMCQ Recall

408.an old pt after surgery uneventfull recoveryafter five-six days suddenly developed chest pain and severe short of breath. What the initiation step.

a) CXRB) CTPAC) ECGd) Cardiac enzymes

409. a lady 30 y old with 4 weeks amenorrhea and clear vaginal discharge presented to ER what is the initial step.

a) USGb) Antibioticsc) BHcGd) vaginal exam

410. pic of red eye with watery discharge , two weeks earlier pt had oral and genital ulcers what the rx

a) antibioticsb) chlormphencol ointment c) oral steroids d)steroid drops

411.Q on leukaemia with blast cells 412.a pt after being drinking and sudden pain in and vomiting in the epigastria area and shock, the pain came after bout of vomiting what is mx.

a) endoscopy b) gastro graffinc) Ct chest

413. a young male came as one of his friends recently has been diagnosed with the testicular cancer. You examined him and found that one testicle is slightly large the other no other abnormality detected. How will you advice him.

a) reassure him as nothing is wrongb) advice him monthly self testicular examinationc) annual AFPd) regular review with USG

414.A pregnant lady with typical migrant came to you what is best rx you give her.a) I/m pethadene b) Morphine and metaclapramidec) morphined) paracetamol and codine

415. ct of intra cerebral bleed dx asked416.typical Presentation of ITP initial test asked

1) FBC 2) Apttc) INrd) bleeding time

417.which of the following is true regarding Naltrexone A. Short acting b. Given for chronic marijuana usersc. Ideal for IV drug user since it is long acting.d. A substitute for Methadonee) . Generic name for naloxone

Page 44: Feb 2012 AMCQ Recall

418.A middle aged women with calf pain from few days came to hospital. her investigations showed Deep vein thrombosis. What is the management?

a. IP management with compression with compression socking and LMWHb. IP management with elevation and LMWHc. OP management with compression and LMWHd. OP management with leg elevation and LMWH

419. Middle aged man with history of cough.CXR showed, lower lobe consolidation. What is the management?

a. oral amoxicillin and clavulanateb. oral azithromycinc.IV ampicillind. Oral flucloxacillin

420.All are S/E of corticosteroid therapy except: - hirsutism- ecchimosis- leucocytosis- osteomalasia- lymphopenia

421. IN a patient with GBS which is ah appropriate way to monitor his oxygenation?a) Forced vital capacity in 1secb) forced expiratory capacity in 1 secc) blood gasesd) pulse oxymetere) Oxygen saturation

422.The mean Bp of a population is 115 mmhg what would be the rage for 95% CI to include 95% population in it with a slandered deviation of 15.

a) 150- 175b) 100- 130c) 70-160d 85-145

423.you are planning to conduct a study what would be the best method to sample the study participants.

a) Take study sample from universityb) Take study sample from your hospital c) Take study sample from old housed )Take study sample the register of general population E) Snowball sampling

424 A 14 yr old boy separated from his parents n currently living w his father since 3 months having bloody diarrhoea .no fever .what is the best investigation to come to a diagnosis?

a) colonoscopy b) sigmoidoscopy c)celiac serology d)FBC

Page 45: Feb 2012 AMCQ Recall

425.A 2 week old child with temp of 40'c came to hospital w hx of seizure + looking ill. Mother very concerned as child not feeding well. What is the diagnosis?

a) febrile convulsion b) septicaemia c) encephalitis d)epilepsy

426.A man presented with the painful defecation and blood on stools. Pain persist for almost 30 min after he opens bowls. What the MX?

a) DREb) colonoscopyc) high fibre dietd) high fibre diet and glyceraltrinitrate ointment e) Steroid ointment

427.What is the characteristic sign of chronic limb ischemiaa ) pain relieved by walkingb) ruborc ) color of limb improved after hanging limb at end of bedd ) numbness

428. a girl is coming to the gp for evaluation with her parents as she is unable to concentrate on her studies. Although she go out with her friends, she dose not seem to enjoy much of the activities, she also experiences some sleep disturbance, which of the following piece of information in the history is most helpful to reach a diagnosis.

a) mother also experience sleep disturbanceb) Brother has ADHDc) She has suicidal ideationd) She use drugs at schoole) She is being bullied at school

429. Bacterial meningitis, viral meningitis, encephalitis please read all about them.430. History of eye injury blowout what will you find in the signs

a) diplopia b) hyphemiac) fracture zygoma

431. SCC of lung and mesothealeoma do these topic

432.Diabetic pt on metformin, endepamide having hypoglycaemic attacks what’s the reason for hypoglycaemia

a) metformin b) endepmide

433.another q on diabetes controlled at the moment but HBA1c increased 7.5 asked the rx, cannot remember the options.74) Q on sodium low 120 asked the reason can’t remember the full Q.

434. A 12 years old boy h/o Nephrotic syndrome not responding well to high dose cortico-steroid therapy, urine protein-++,hypertension, hematuria with red casts. WOF can be the cause:

A .Ig A nephropathyB .PSGNC .focal glomerulosclerosis

Page 46: Feb 2012 AMCQ Recall

435. A 2 yrs old child , parents complaining of his loss of wt for few days with abd. distension and Diarrhoea and failure to thrive after weaning started . WOF is the DX?a. Cystic fibrosisb.Coeliac diseasec.Whipple disease

436. Stem abt Kawasaki disease given and asking abt treatment.

Other recalls1. A patient with Duke C colon cancer, had left Hemicolectomy. For 5 year survival rate, what can be done?a. Total Colectomyb. Abdominal radiationc. 5-fluorouracil

2. A picture showing whitish aspirate from parotid ducta. Parotitisb. Calculus

3. Whitish echogenic lesion on mammography picture of 60 y.o. woman. She admits she had an accident of hittingwith a steering wheel on the same breast a year ago. What is the likely diagnosis?a. Focal fat necrosisb. Carcinoma of breast

4. Fundoscopy pictures of central retinal artery occlusion/ central retinal vein occlusion5. 50 y.o. man complains of flashes and floaters in visiona. retinal detachment

6. CT picture of pulmonary embolism. In the history, patient has a shortness of breath. What is the treatment?a. Aspirinb. Warfarinc. Heparin

7. A patient was found to have Peau de orange while taking an ECG. What is the diagnosis?a. Carcinoma breast

8. A picture of venous ulcer on the medial malleolus of leg. What is the cause?a. Previous tibia/fibula fractureb. Arterial insufficiency

9. A picture showing ulcer near thenar eminence of hand?a. Epitheliod sarcoma

10. A patient diagnosed with Sigmoid volvulus. What is the feature suggestive of ischemic change?a. gross distention of abdomenb. 10 cm dilatation of sigmoidc. focal paind. colicky pain

Page 47: Feb 2012 AMCQ Recall

11. A 45 y.o. male presented with swollen inguinal lymph node. Where could be the primary?a. Rectumb. Testisc. anal canald. colon

12. A patient came in with left eye pain of 6 min duration? comes for about 1 hour every day, now increasing intensity,fundoscopy shows arterial injection. What is the treatment?a. Acetazolamideb. Hydrocortisonec. topical pilocarpine

13. A 20 y.o. female presented with malaria rash, joint pain. What test will you do?a. ANAb. Complement assayc. Anti DS DNA

14. M.I. patient, ECG showed ST elevation in V2,V3,V4,V5. Tertiary hospital 300 km away. Patient was given oxygen,morphine, aspirin. What should you do?a. PCI ( percutaneous coronary intervention)b. t-PAc. streptokinase

15. A 15 y.o. patient from Sudan found to have a low calcium. What will you give him?a. Calcium tabletb. Vitamin Dc. Cholecalciferol

16. A patient has been a chronic carrier of Hepatitis B for 6 months, What will you find on him?a. HBS Agb. HBS Abc. HBE Agd. circulating DNA

17. A boy had hepatitis A infection and 10 days after he became physically fit. When can he go back to school?a. Now and use separate toiletb. after 3 daysc. once the enzyme comes to normal

18.Who is likely to get Coxiella burnetti infection?a. Dairy workersb. fresh water fisherman

19. A lady presented to EMD in a drunk state with a black around eye. What is the most important thing to rule out?a. Fallb. Alcoholc. Domestic violence

Page 48: Feb 2012 AMCQ Recall

20. A patient with right upper quadrant pain, mild jaundice, all enzymes elevated, bilirubin 40. What is the diagnosis?a. Cholangitisb. Acute cholelithiasisc. hepatitis

21. A patient was given Morphine. What will be the blood gas analysis picture?a. Respiratory acidosisb. respiratory alkalosis

22. A patient had bitemporal hemianopia. Where is the lesion?a. Optic tractb. Optic Chiasmac. Optic nerve

23. Question on Osteoporosis24. A 40 y.o. female came with history of food getting stuck in the throat, regurgitation of undigested food. What islikely diagnosis?a. Carcinoma Oesophagusb. Zenker Diverticulum

25. 17 years old jogging then collapsed.somebody did CPR and was revived when she arrived at ER. Has history thatfather died suddenly. Which is the likelydiagnosis?--HOCM

26. ECG on V tach. Identify and Treatment--Amiodarone

27. Woman when she defecates a mass is noted to bulge out of her introitus. Most likely diagnosis?---Rectocoele

27. question about painless hematuriaa. bladder tumorb. renal tumor

28. question about a man 35 years old asking about his risk of developing prostate CA. Father diagnosed prostate CAat 85 years old.a. reassureb. PSAc. USGd. DRE annually

29. Old man was found by police and brought to hospital. Daughter has many complaints about father. She said hehas been living in squalor and eating rubbish.what kind of dementia?a. frontal lobe dementiab. Lewy body dementia

30. question july 2010 #10131. question july 2010 #110

Page 49: Feb 2012 AMCQ Recall

32. question july 2010 #11133. question about an old lady after closing the window (?) sustained a crushed fracture on her vertebrae. She had ahistory of operation for CA of caecuma. metastasis from primary cancerb. osteoporosis

34. patient just had Cricothyroidectomy and developed stridor while she was still in the recovery room. What will youdo?a. call the surgeonb. explore the wound in the wardc. remove the skin sutures

35. Xray given. Man is a non-smoker and working in the mine industry (did mention the years,think it was more than20years). Has been coughing.--mesothelioma

36. question about percentage passing the disease to their children with cystic fibrosis-- 0%

37. Xray given. Man coughing, smoker and working in the mine industry. What is the most probable diagnosis?--bronchogenic CA

38. Naloxone is:a. short acting

39. X-ray of hip osteopenia ALP is 800 ( that is the only information given ) Rxa. Biphosphonateb. Calciumc.Vitamin b12d. Radiotherapy

40. Pic of both lower limbs showing red rash. 34 year old male with diarrhoea and admitted in the hospital Dx?- HSP

41. CXR – night sweat, dry cough, wt loss, enlarge bilateral cervical lymph nodes. What is the likely diagnosis?a. Sarcoidosisb. Tbc. Lymphomad. CLL

42. A patient who started hydrochlorothiazide with digoxin develop nausea electrolyte pic given only potassiumdecreasea. Stop digoxin and give K supplementb. Stop hydrochlorothiazide and give K supplement

43. SVT ECG. Treatment?a. Adenosineb. Amiodarone

Page 50: Feb 2012 AMCQ Recall

c. Verapamild. Digoxin

44. ECG . a pt who had stroke came to hospital. on 5th day, she collapsed in the toilet. BP pulse?? What will you donext to diagnose his condition?a. CTPAb. Echo cardio

45. A boy came with bloody diarrhoea with less urine output, what is the diagnosis?a. Renal failureb. HUS

47. A pt who had an injury to eye due to hit from metallic spike, what will you do?• Use local anaesthetic and removed

48. A lady and husband came to you recently dx cystic fibrosis concern about their child getting ita. 0.25 chanceb. 0.5 chance

49. A couple comes telling that one child diagnosed as cystic fibrosis concern about next child incidencea. 0.5b. 0.25c. 100

51. CT head show hemorrhage. Pt on warfarin and INR 4.5 what will you do?a. give FFPb. give platelet

52. CT head showing same lesion like infarctiona. Multi infarct dementiab. Cerebral hemorrhagec. Cerebral abscess

54. VT ecga. Adenosineb. Amiodaronec. Adrenaline

55. A lady complain loss of finger grip in index and thumb what nerve is affected?a. Radial nerveb. Ant. Interosseousc. Ulnar nerve

57. 50 year old man coming with scrotal pain which is reddened, what will you do?a. Urine catheterb. blood culturec. FNAC

58. Fundoscopy pic, HTN, polyuria, polydipsia, what is dx?a. DMb. HTNc. CRAO

Page 51: Feb 2012 AMCQ Recall

d. CRVO

59. A male who used to work in mine industry for 20 years and smoker for 40 years. What is the dx? ( CXR given)a. Small cell carcinomab. Squamous cell carcinomac. Mesothelioma

60. A chinese lady comes to your clinic. Husband is recently diagnosed having haemochromatosis. They have 9 y.o.and 18 year old children . Wife is notscreen for haemochromatosis. What will you do?a. Check wife onlyb. Check 9 y.o. onlyc. Check 18 year old onlyd. Check both children

71. A down syndrome underwent surgery and developed hypersensitivity. What is the cause?a. Latexb. Anaesthetic drugs72. A 34 y.o. male having ACE inhibitor, Statin, b-blocker develop calf pain, tenderness in calf. What is the next step?a. Check CK levelb. Check creatininec. Check FBC

73. A pic both eye increase tearing. What will you do?a. Refer to plastic surgeryb. Cold compression

74. CXR given and ask of diagnosisa. Pulmonary oedemab. Metastasisc. Carcinoma

75. At 2nd Post Op day, a person developed agitation and confusion. Temp normal, pulse and BP normal. What is thecause?a. Alcohol withdrawalb. Pulmonary atelectasisc. Electrolyte imbalance

76. FUNDOSCOPY PICTURE ( + microaneursym seen) : patient has a history of Diabetes Mellitus type 2 later onpositive blurring of vision and soughtcheck up. What is the likely cause?a. Diabetic retinopathyb. hypertensive retinopathyc. retinal detachmentd. macular degeneration

77. FUNDOSCOPY PICTURE ( + drusen): 78 y.o. woman had a gradual onset of bilateral visual loss. What is thediagnosis?a. CRVO

Page 52: Feb 2012 AMCQ Recall

b. CRAOc. Diabetic retinopathyd. Retinal detachmente. Macular degeneration

78. A 60 + y.o. man presents with cough and productive sputum, weight loss of 3 kg ( forgot the duration). He had ahistory of working in a gold mine industryand was diagnosed before of moderate silicosis. The patient is a NON-SMOKER. What is likely the diagnosis ofpresent symptom?a. mesotheliomab. exacerbation of silicosisc. pulmonary tuberculosisd. bronchogenic carcinoma

79. A picture of chest X-ray and ECG in one stem. A patient with a history of ischemic heart disease and hiatus herniaattends a dinner conference. Aftereating, he suddenly developed retrosternal pain and collapse. He was rushed to ED and given oxygen and iv fluids.What is the next management?a.Pulmonary angiographyb. Nasogastric decompressionc. Chest tube drainaged. Emergent laparotomye. Ultrasonography( no options for ECG)

80. a 70 y.o. woman diagnosed of cancer of the caecum. What is the first symptom noted in this patient?a. palpable massb. altered bowel habitsc. tiredness and fatigued. bowel obstructione. dyspepsia

81. A 30 y.o. man with multiple rib fractures following a motor vehicle accident. He is noted of having paradoxicalbreathing. What would be the mostappropriate next step in management?a. strap the chest wallb. controlled and positive ventilationc. insert chest tubed. 100 % oxygen via bag mask

82. Old lady comes to the ED following MVA with a wrist fracture and confusion. What would be the most appropriatenext step?a. Urgent CT headb. Xray wristc. Wrist plaster cast

Page 53: Feb 2012 AMCQ Recall

93. A 55 y.o. man comes to your clinic and concerned of his family history which his father was diagnosed of havingprostate cancer at 85 y.o. and neighborwas diagnosed of same condition recently. He is worried he might get prostate cancer. On DRE, his prostate wassmooth, soft and median sulcus was felt. Noother symptoms. What would you advice?a. reassuranceb. PSA for the next 4 weeksc. DRE annuallyd. Ultrasound

94. Patient has an ulcer at medial malleolus of left leg. What could be the risk factor?a. smokingb. previous history of DVT at same legc. diabetes mellitusd. malignant change

95. Man had lower limb injury involving the fat layer. He had a complete vaccination history and last dose was 9 yearsago. What is the most appropriatemanagement?a. aDTPb. aDTP and Igc. Igd. reassure

96. A 27 y.o. man had a major motor vehicular accident which liver and spleen have lacerations. multiple wounds andfractures. He received 15 units of bloodduring resuscitation. Few hours after, it was noted positive bleeding in all sites. What is the cause?a. transfusion reactionb. hypocalcemiac. spleen ruptured. DICe. dilutional thrombocytopenia

97. A young patient had a fall injury. PO2 60 and PCO2 55. Patient has breathlessness but conscious. Whats the nextstep management?a. paralyze, intubate and ventilateb. 100% oxygen via bag maskc. controlled oxygend. Chest xraye. chest tube and drain

98. A patient is diabetic and was scratched by a cat in the arm and few days later, + red rash at arm, forearm andanterior shoulder ( no LAD, No fevermentioned in the stem). What is the likely diagnosis?a. Cat scratch feverb. Cellulitis

Page 54: Feb 2012 AMCQ Recall

99. A boy with long ears and large forehead. ( this is a long stem mentioning uncle also has same features) What isthe diagnosis?a. Down syndromeb. XYYc. Klinefeltersd. Fragile X syndrome

100. What is the chance of having a child affected of cystic fibrosis if one parent is a carrier?a. 0b. 25c. 50101. A pedigree pattern of Autosomal recessive.

102. Picture : exactly the same page 206 anthology of medical condition. what is the diagnosis?a. tinea capitisb. pityriasis capitis ( dandruff)c. pityriasis versicolord. seborrheic dermatitis

103. . Old patient has history of syncope 3 episodes. Choose ECG that fits:a. ecg of Atrial fibrillationb. ecg of Atrial flutterc. ecg of ventricular ectopic beatsd. ecg of Complete heart blocke. ecg with tall t waves ( Hyperkalaemia?)

104. Which one of the following studies is LEAST useful?a. Cohortb. case controlc. cross sectionald. case reporte. systematic review

105. A picture of psoariasis at both lower legs with extensive lesions. What is the treatment of choice?a. UVB lightb. calcipotriolc. steroids( no dithranol, no methotrexate, no tar in the options)

106. A long stem about patient has dryness in the eyes and mouth. Anti RO and Anti La POSITIVE detected. Whatsthe management?a. Hydroxychloroquineb. Steroidsc. Methotrexate

107. A patient was diagnosed of having colon cancer ( i forgot the stage) , surgical operation done and chemotherapyshould be started. whats themanagement?a. 5 Fluorouracilb. Cisplatin

Page 55: Feb 2012 AMCQ Recall

c. Methotrexate

108. A picture of patient having redness in the eye ( anterior uveitis). He had a history of mouth ulcer and genitalulcers. Whats the treatment?a. topical hydrocortisoneb. oral steroids

109. WOF features would be absent in pseudobulbar palsy?a. dysarthriab. dysphagiac. absent gag reflex

110. A 24 y.o. lady has one episode of seizure. She experienced of having muscle jerks in her right arm few monthsago. She asks you on advice for driving?a. can drive after 6 monthsb. can drive after 3 monthsc. can drive after 12 monthsd. can drive after 24 months

111. Patient with a maintenance of digoxin for AF and thiazide for HTN. Recently, she complains of nausea, vomiting.and palpitations. What would be themost appropriate step?a. stop digoxin and give potassium supplementsb. stop thiazide and give potassiumc. give potassium

112. Chest X-ray showing a cavitary lesion at the upper right lobe. Patient came from Sudan. He comes with fever andnight sweats. Sputum negative for acidfast bacilli. What would be the most appropriate management?a. Isoniazid prophylaxisb. Isoniazid, rifampicin, pyrazinamide and ethambutolc. Amoxicillind. Ceftriaxonee. no treatment necessary

113. A driver from interstate has moved to your locality recently and is stable on lithium for past 20 years. How do youmonitor?a. check lithium every 3 monthsb. check lithium every 6 monthsc. check lithium every 1 year

114.85 y.o. woman presented with right thigh and leg pain after fall. Investigations are: calcium 3.5 ( 2.1-2.6), Urea- 10( phosphate level not given). Mostcommon cause?a. Osteoporosisb. Vitamin D toxicityc. Primary hyperparathyroidismd. Hypercalcemia of malignancy

Page 56: Feb 2012 AMCQ Recall

115. A middle aged woman with calf pain and tenderness from few days came to hospital. Her investigations showedDVT. She has no other associatedsymptoms. What is the management?a. inpatient management with compression stocking and LMWHb. IP management with elevation and LMWHc. outpatient management with compression and LMWHd. OP management with leg elevation and LMWH

116. Middle aged man complains of cough and productive sputum. CXR showed patchy infiltrates. What is themanagement?a. Oral amoxicillin and clavulanateb. Iv ampicillinc. oral flucloxacillind. Oral azithromycine. tetracycline

117. Young girl collapsed in a party brought to ED. She is confused. fever of 40 deg C. Noted muscle jerks. BP:157/98. What is the initial management?a. Thiamine and oral hydrationb. cold saline infusion and cold blanketc. diazepamd. haloperidol

118. ALL are side effect of steroids EXCEPT:a. hirsutismb. ecchymosisc. osteomalaciad. lymphopeniae. increase WCC

119. 65 y.o. man had a headache with tenderness on the scalp especially on the right side. Positive history ofheadaches for few days. What is the nextmanagement?a. temporal biopsyb. oral steroidsc. CT scan( no ESR or FBC options)

120. In a patient with GBS which is appropriate way to monitor his oxygenation?a. forced vital capacityb. ABGc. pulse oximeterd. O2 saturation

121. A man complains of cough productive with green sputum and occasional blood. No fever. WOF is the best longterm management?a. low dose tetracycline for 12 monthsb. anti tb drugsc. postural drainaged. reassure

Page 57: Feb 2012 AMCQ Recall

122. Patient has a right LAD when biopsy + squamous cell ca. Where is the primary site?a. breastb. thyroidc. bronchusd. kidney

123. Toxic shock syndrome caused by staphylococcus aureus is associated with:a. super absorbent sanitary padsb. endotoxin production

124. Construction worker complains of eye redness and lacrimation after hammering a nail at work. + metal spikesseen. Whats is your management?a. antibioticsb. irrigation of fluidsc. local anesthetic and probe the eye

125. an indigenous boy complains of greenish discharge coming out from the ear and also has green nasalsecretions. What is the appropriate management?a. swab ear dischargeb. swab nasal dischargec. audiometry( no option antibiotic)

126. An alcoholic man with pain radiating to the back. Diagnosed of acute pancreatitis. WOF is the LEAST likely topresent with?a. constant painb. nocturiac. diarrhoead. regurgitation of food

127. A patient complains of lower back pain. On PE noted lumbar spine is not in its normal curvature. NO numbnessor decrease ankle jerk. What to do next?a. lumbosacral xrayb. MRIc. Nerve conduction study

128. A patient came in with haematuria, dysuria. Urinalysis showed many rbc and atypical nuclei. What to do next?a. renal ultrasoundb. ct scan of abdomenc. urine culture and sensitivity

129. An elderly patient brought to ED with lightheadedness and palpitations. was diagnosed of Aortic stenosis. ECG isgiven. What is the management?a. adenosineb. digoxinc. verapamild. not to repeat the carotid massage

Page 58: Feb 2012 AMCQ Recall

130. A 42 y.o. Nulliparous woman has a bulging mass in the perineum especially during straining on defecation. Whatis the likely cause?a. cystocoeleb. rectocoele

131. A 38 y.o. man was attending a party and ate some peanuts in the cake, suddenly had hoarseness of voice anddifficulty of breathing. Whats theimmediate management?a. Adrenaline IMb. intubate and ventilatec. Antihistamined. administer Oxygen

132. A 50 y.o. woman complains of pain at maxillary area and some redness. ( no rashes mentioned). What is theappropriate management?a. Carbamazepineb. Acyclovirc. Famciclovir

133. A GP is in rural area, you notice that recently more cases of Hepatitis C has been diagnosed and want to doresearch to find out the incidence ofhepatitis C in the community with a population of around 3000. How will you get the information you need for theresearch?a. Find the number of all patients with current hepatitis C antibodyb. All patients with current hepatitis C PCR positivec. All patients with current Hepatitis C antigen positived. All cases diagnosed as hepatitis C positive for the past 5 years

134. A man with heavy alcohol intake brought to ED. BP 180/100 HR 120, + tremors ( fine) bilateral in both hands. Noconfusion, not agitated. What is theappropriate management?a. Propanololb. Benztropinec. Diazepamd. Haloperidol

135.Naltrexone is:a. short actingb. easily absorbed orallyc. substitute for methadone in treating drug dependence

136. A 20 y.o. presented with weakness and oedema of left hand with history of cat scratch 4 days ago. Lymph nodeenlarged and non tender. Whats thecause?a. cellulitisb. cat scratchc. axillary vein thrombosis

Page 59: Feb 2012 AMCQ Recall

137. A patient cannot extend wrist and fingers. All other normal. What nerve is affected?a. anterior interosseous nerveb. ulnar nervec. radial nerved. posterior interosseous nerve

138. A 23 y.o. Nurse working in Rural hospital presented with cough followed by hemoptysis for 1 month. CXR given( apical lobe affected)What is the diagnosis?a. HIVb. Tuberculosisc. Lung CAd. Pneumonia

139. A bacterial conjunctivitis picture given. What is the treatment?a. symptomaticb. ciprofloxacin dropsc. chloramphenicol drops

140. A man with symptoms of difficulty in plantar flexion, inversion, dorsiflexion, eversion of foot. Knee reflex ispresent. Where is the lesion?a. Sciatic nerveb. L5c. Tibial nerved. Common peroneal nerve

141. CT SCAN of cerebral hemorrhage and cerebral tumour

142. A question on Heparin induced thrombocytopenia. The patient is on low molecular weight heparin. What will youdo next?a. change to unfractionated heparinb. change to another anticoagulant other than heparin group

143. A 75 y.o. with complains of bloating, abdominal distention, diarrhea alternating with constipation. No blood instools. Colonoscopy done 12 months agoand came out normal. What to do next?a. repeat colonoscopyb. diet advice

144. A 75 y.o. presented with postprandial abdominal pain. No other assoc. symptoms. What is the diagnosis?a. Carcinoma duodenumb. Carcinoma cardiac. cholecystitis

145. 4 other questions on GIT146. typical question on Bells Palsy

147. Management of Tennis elbowa. no option for restb. elbow immobilisation splint

Page 60: Feb 2012 AMCQ Recall

c. brace below elbow

148. Biker fell from bicycle 1 week back. No history of unconsciousness. But now presenting with occipital headache.What is the diagnosis?a. Epidural hemorrhageb. Subdural hemorrhagec. Post traumatic headache

149. ECG of WPW syndrome with wide QRS. What is the management?a. Amiodaroneb. Adenosinec. Cardioversion

150. Sign of acute limb ischemia indicative for surgery?a. Painb. paralysisc. color

151. Sign of chronic limb ischemia?a. Painb. paralysisc. rubor

152. A picture of SCC. what will happen next?a. involutesb. grows fastc. spread to lymph nodes

153. A patient is on Metformin, Perindopril and diuretic. Amoxicillin was taken 3 days ago for URTI. He developedswelling and itching of right side of tongue.what is the cause?a. Amoxicillinb. Perindoprilc. Diuretic

154. Treatment of Alopecia Areata?a. application of hydrocortisone on that areab. injection of steroid on that area

155. MORE questions on Peptic Ulcer disease, CA duodenum, CA cardia156. An old man was well before and suddenly confused and agitated with memory loss in 2 weeks time thoughts onand off. MMSE score 26/30 brought byhis family as symptom getting worse. What is the diagnosis?a. dementiab. delirium

157. A lady is hepatitis B positive. She is coming for advice regarding the Hepatitis B transmission to her unborn baby.What will you advice her?a. She should have CS for deliveryb. She should not breastfeed her child as risk is greatc. the baby should receive Hepatitis B immunoglobulin and vaccination at delivery

Page 61: Feb 2012 AMCQ Recall

158. A 35 y.o. lady with mother having breast cancer at 47 years and sister diagnosed with ovarian cancer at 30 years.What is her percentage of developingbreast cancer?a. 0.25b.0.10c. 0.50d. 0

159. What is the risk of conceiving a child with Down syndrome for a 38 y.o. female?a. 1:10b. 1:100c. 1:200d. 1:400e 1:700

160. A 19 y.o. boy complaining of having difficulties in playing sports at school. His height is at 97th percentile for hisage and weight is at 3rd percentile. Whatis your next step management?a. Karyotypingb. Bone scanc. Growth hormoned. CT scane. Cardiac ultrasound

161.A patient who worked in a gold mine for 20 years and is a smoker and history of working in petrol station for 20years now presents with hematemesisand cough. Xray was given. How would you diagnose him?a. bronchoscopeb. USGc. CT scand. needle aspiration

162. A middle aged lady with history of pain over distal fingers, more on palpation with no obvious deformity for 20years, +anaemia. CXR shows enlargedheart . What is the diagnosis?a. Sclerodermab. SLE

163. A picture of SCC at lower lip, what is the most common cause?a. smokingb. sun exposurec. metastasisd. unknown cause

164. A patient had an accident, his pulse is 120, BP is 120/80 lying down and 90/70 standing. What is the percentageof blood loss?a. 10%b. 20%c. 25%d. 35%

Page 62: Feb 2012 AMCQ Recall

e. 40%

165. Old patient at nursing home with on and off constipation. Xray of the abdomen ( Blue book page 458). What isthe diagnosis?a. sigmoid volvulusb. large bowel obstructionc. small bowel obstructiond. adhesion

166. ECG of WPW. Asking for treatment?a. carotid massageb. amiodaronec.digoxind. adenosine

167. ECG of an old lady with SVT and then suddenly asystole after carotid massage. What to do next?a. amiodaroneb. adenosinec. shift her to ICUd. atropinee. do not do carotid massage again

168.Venous ulcer asked about the most common association?a. smokingb. dvtc. long bone fracture

169.A picture of ulcer of the medial malleolus. What is the diagnosis?a. arterial ulcerb. venous ulcerc. diabetic ulcer

170.A patient complain of jaundice with dark urine and pale stools. Pain in right upper quadrant. There is a masswhich moves with respiration. Most likelydiagnosis?a. choledocholithiasisb. carcinoma of ampulla of pancreasc. mucocoele

171. An old patient who had drug eluting at prosthetic valve place, clopidogrel. and came to you for preoperative clinicevaluation. He has non complicatedvaricocoele. What will you advice him regarding the operation?a. reassess the need for surgeryb. give FFP before surgeryc. stop Clopidogrel few days before surgeryd. Give vitamin K

172. A lady of a certain age had Subtotal thyroidectomy several hours ago. She was noted to have stertorousrespiration in the ward. There is some swellingand redness in the post op site. What is the immediate management?

Page 63: Feb 2012 AMCQ Recall

a. perform bedside ultrasoundb. remove the deep muscle suturesc. call the surgeon and arrange intra op exploration of the wound sited. request CXR urgentlye. remove the skin staples

173. A 60 y.o. man with a long history of diabetes comes to the hospital with ulcer in each dorsum of the toe in bothlegs. There is foul smelling dischargefrom the right side ulcer. What would be the most appropriate next step in management?a. amputate the toesb. do doppler ultrasoundc. give antibiotics ( amoxicillin and clavulanate)

174. A 30 y.o. man with multiple rib fractures following a motor vehicular accident. He's having a paradoxicalbreathing. What would be the most appropriatenext step in management?a. strap the chest wallb. CPAP with ventilation

175. A picture of an old lady with a red swollen cheek like cellulitis. Whats is the management?a. incision and drainageb. antibioticc. OPG

176. A teenage boy came with pain in his right knee after knee injury in the sport game. His knee is red and swollen,hot, painful. What is the investigation?a. CTb. MRIc. aspiration of the kneed. antibiotic

177. An old patient after uneventful recovery from surgery, 5 or 6 days post op, suddenly developed chest pain andsevere short of breath. What is the initialstep?a. CXRb. CTPAc. ECGd. Cardiac enzymes

178. A picture of red eye with watery discharge, 2 weeks earlier, patient has oral and genital ulcers. What is thetreatment?a. antibioticsb. chloramphenicol ointmentc. oral steroidsd. steroid drops

179. Question on leukemia with blast cells180. A patient after binged drinking and sudden vomiting and pain in the epigastric area later on, + shock. What is the

Page 64: Feb 2012 AMCQ Recall

management?a. Endoscopyb. Gastro graffinc. CT chest

181. A young male came to you asking for advice because one of his friends has been diagnosed with testicularcancer recently. You examined him andfound out that one of the testicle is slightly larger than the other. No other abnormality detected. What will you advicehim?a. reassure him as nothing is wrongb. advice him monthly self testicular examinationc. annual AFPd. regular review with USG

182. CT scan ( looks like intra cerebral bleed) What is the diagnosis?183. Typical presentation of ITP. what is the initial test?a. FBCb. APTTc. INRd. bleeding time

184. WOF is true regarding Naltrexone?a. short actingb. given for chronic marijuana usersc. ideal for IV drug user since it is long actingd. a substitute for methadonee. generic name for Naloxone

185. Middle aged man with history of cough. CXR showed lower lobe consolidation. What is the management?a. oral amoxicillin and clavulanateb. oral azithromycinc. IV ampicillind. Oral Flucloxacillin

186. In a patient with GBS which is an appropriate to monitor his oxygenation?a. forced vital capacity in 1 secb. forced expiratory capacity in 1 secc. blood gasesd. pulse oximeter

187. A 14 y.o. boy separated from his parents and currently living with his father since 3 months having bloodydiarrhea. No fever. What is the bestinvestigation to come to a diagnosis?a. colonoscopyb. sigmoidoscopyc. coeliac serologyd. FBC

188. A man presented with the painful defecation and blood on stools. Pain persist for almost 30 mins. after he openshis bowel. What is the management?

Page 65: Feb 2012 AMCQ Recall

a. DREb. colonoscopyc. high fibre dietd. high fibre diet and glyceryl nitrate ointmente. steroid ointment

189. What is the characteristic sign of a chronic limb ischemia?a. pain relieved by walkingb. ruborc. color of the limb improved after hanging limb at the end of the bedd. numbness

190. Bacterial meningitis, viral meningitis, encephalitis- please read all these topics191. History of eye injury, blowout injury. What will you find?a. diplopiab. hyphemac. fracture zygoma

192. SCC of lung and mesothelioma- please read these topics193. Diabetic patient on Metformin, Indapamide having hypoglycemic episodes.What is the reason for thehypoglycemia?a. Metforminb. Indapamide

194. Another question on Diabetes and is controlled at the moment But later on HBA1C increased to 7.5. Asked forthe treatment?( cannot remember the options)

195. A case of hyponatraemia ( Na- 120). Stem asked for the reason. ( can't remember the full question and options)196. Question on intestinal obstruction197. 3 ECG questions, 3 CT scan, 2 CXR, 4 skin condition with pics.198. What is true about HIV?a. Antibodies appear after 9 yearsb. Risk transmission to foetus is 50%c. Ratio CD4 to CD8 is more than 5

199. A middle aged lady with history of pain over distal fingers more on palpation with no obvious deformity for 20years , + anaemia, CXR shows enlarged heart. What is the diagnosis?a. sclerodermab. SLE

200. A man post gastrectomy 3 years ago complains of lethargy. On examination, + pallor. Blood Investigation: Hb 9.0,MCV 110. All other normal range. What is the diagnosis?a. Iron deficiency anaemiab. vitamin b 12 deficiency anaemiac. Chronic anaemia

201. A woman with history of passing small quantities of urine after bouts of cough each time was sent forinvestigation. What is the best investigation to come to a diagnosis?

Page 66: Feb 2012 AMCQ Recall

a. CT abdomen and pelvisb. Laparoscopyc. Cystoscopy

202. A 30 y.o. man was scratched by his cat and 7 days after, there were pustules from humerus to thumb andoedema of arm at the site of the scratch. What could be the diagnosis?a. cellulitisb. cat scratch feverc. Herpes

203. Inferior wall MI of a 55 y.o. man with hx of HTN. Patient is at Metropolitan hospital. Patient has been givenoxygen, nitrates, morphine and aspirin. Patient came to hospital after 4 hrs of symptoms. What would be the mostappropriate next step in the management of this patient?a. check troponin right now and 8 hours laterb. coronary angiography

204. A 30 y.o. woman comes to your clinic with complaints of a seizure like activity which occurred last night. She saidher seizures were well controlled the past 3 years with antiepileptic medication and again it reappeared since recently.How long should she wait to drive?a. 6 months seizure free periodb. 2 years seizure free periodc. she can drive straight away

205. A 30 y.o. lady comes with a bruise on the left cheek and a cut on the lip. If you check, which system would you beable to come to a diagnosis?a. CVSb. Respiratory systemc. Central Nervous systemd. Musculo skeletal system

206. An HIV patient came to your clinic along with partner and asking for the best prophylactic measures they shouldtake:a. Use condom and safe sexb. both should use anti HIV drugs

207. Picture of man with normal eyes. History of watery discharge. What is the management?a. Chloramphenicol ointmentb. cold compressc. Acetazolamided. acyclovir drops

208. 6th nerve palsy picture given. Diagnosis?209. Amitryptilline intoxication. What is the management?a. IV fluidsb. intubation and sodium bicarbonatec. gastric lavaged. activated charcoal

Page 67: Feb 2012 AMCQ Recall

210. COPD patient received 6 L of oxygen and get worse. What is the next step?a. Immediate CXRb. reduce oxygenc. increase oxygend. CPAP

211. A patient is on following medications, thiazide 25 mg, Beta blocker, presented with polyuria, polydipsia. His bloodsugar level is 17 mmol/L. What is the next management?a. Stop Thiazide and reassessb. Start on oral hypoglycemicc. insulin treatment

212. A patient with severe chest pain with half an hour history. ECG was done which was normal, cardiac markerswere normal too. What is the most appropriate step in management of this patient?a. send him homeb. give GTNc. do PCId. stress test

213. A young slim marathon runner with a history of 2 hour sudden onset of left sided chest pain and shortness ofbreath on mild exertion. Diagnosis?a. MIb. spontaneous pneumothoraxc. cardiomyopathy

214. A 35 y.o. woman with h/o headache and bilateral homonymous hemianopia. She recovered from headache butthe visual defect still persists after 6 wks. She's asking when she can drive again?a. after 3 monthsb. after 6 monthsc. never drive againd. after 1 year

215. A young lady presents to GP early morning with history of 8 glasses of spirits last night, with history of nauseasince last night. In the morning, still positive nausea with vomiting. P.E. normal. Next step in management?a. review in 4 hoursb. review later in the dayc. explain this is due to excess alcohol and no need to worryd. prescribe anti-emetics and review if symptoms recur216. Patient with osteoporosis and oesophagitis. What is the treatment?a. alendronateb. zoledronic acidc. increase milk intaked. risedronatee. give calcium supplement

217. WPW syndrome with ventricle ectopic. What is your next step in management?a. adenosineb. verapamilc. amiodarone

Page 68: Feb 2012 AMCQ Recall

218. A picture of keratoacanthoma. What is the diagnosis?

219. Same picture as above. What is the next step in management?a. wide local excision of at least 4 mm marginb. local incisionc. local excision with sentinel node biopsyd. refer to plastic surgerye. review after 6 weeks

220. CT scan showing intracerebral hemorrhage, patient was on warfarin and his INR at the moment is 4.9. Whatwould be the most appropriate next step?a. give vitamin Kb. give protamine sulfatec. give FFP

221. A 50 y.o. man comes with a swelling in the groin region. Its reducible on lying down and patient says its slightlytender. Patient was put on aspirin and clopidogrel 3 months back for atrial fibrillation. What would be the mostappropriate next step in management?a. give vitamin K and send patient immediately for surgeryb. give FFP and surgeryc. stop aspirin and clopidogrel for 1 week then proceed with surgeryd. just refer the patient immediately for surgerye. nothing needs to be done

222. Patient comes to you with complaints of swelling under the jaw and its more prominent after food. Bimanualpalpation was done and what would be the most appropriate next step?a. intraoral X-ray of mandibleb. CT scanc. sialogramd. OPGe. USG

223. History of foreign body in the eye. What is the initial management?a. remove the foreign body with the needleb. local anesthetics drops and irrigate with fluidsc. antibiotic dropsd. miotic drops

224. Patient with earache, swelling in the cheek. On examination, ear is normal. What is the next initial investigation?a. OPGb. Head CTc. Sialogramd. CTe. Xray mandible

225. A 24 y.o. man with severe knee pain. On examination, you found effusion around the knee, + pain on kneemovements. Diagnosis?a. anterior cruciate ligament injuryb. posterior cruciate ligament injury

Page 69: Feb 2012 AMCQ Recall

c. medial meniscus injuryd. lateral meniscus injury

226. Elderly patient with history of anaemia and respectively symptoms of it, no other history or symptoms. What isthe next step investigation?a. gastroscopy and colonoscopyb. sigmoidoscopyc. colonoscopy

227. A patient with 15 % pneumothorax. Management?a. reassurance and observeb. drainage under seal waterc. oxygend. intubatione. aspiration

228. ( Big stem) Old man with many comorbidities, + polypharmacy, falls every morning. Most appropriatemanagement?a. CTb. ECGc. Echo( no BSL option)

229. Carpet layer presents with pain and tenderness in one knee. Treatment?a. corticosteroids intra articularb. rest for 2 weeksc. compression bandage

230. A patient complains of developing digital ulcers when exposed to cold in winter. Treatment?a. Hydroxychloroquineb. Oral prednisolonec. Im prednisolone

231. Picture of Erythema multiforme232. Picture of drug rash on legs233. A patient comes with peripheral weakness and has difficulty seeing at night. What would you give her initially?a. Folic acidb. Iron supplementsc. Vitamin B 12

234. A man with a low visual acuity which becomes worse in sunlight. Pic is given.a. Intraocular lensesb. Phacoemulsificationc. pilocarpined. keratoplastye. acetazolamidef. trabeculectomy

235. A plumber comes with cough. He is smoker. CXR given: + changes in pleura. What is the definitive investigation?a. Bronchoscopy and bronchoalveolar lavage

Page 70: Feb 2012 AMCQ Recall

b. pleural biopsyc. CT scan236. Bone pain, very high ALP, no calcium given. Next IX?a. Calciumb. electrophoresis

237. Nephrotic syndrome becomes steroid resistant. Dx?a. IgA nephropathyb. Post streptococcal GNc. Focal segmental glomerulosclerosis

238. A patient with weakness in one hand and headache. Diagnosis?a. atypical migraineb. migraine

239. A 32 y.o. patient with rash on buttocks, + fever. DX?a. HSPb. ITPc. HUS

240. Fundoscopy 76 y.o. with CRAO

241. For Helicobacter pylori medications: antibiotic with PPI will help to:a. increase the relapse timeb. PPIs help antibiotic to penetrate the ulcer

242. Several syncope in old patient. PE: HR 44 BP 180/110, + murmur. What is the cause?a. Aortic stenosisb. Complete heart block

243. A patient comes with h/o of treatment of osteoarthritis with Celecoxib, patient develops mouth ulcer. Mx?a. cease celecoxibb. give folic acid

244. Viral meningitis. CSF findings?245. Ischemic abdominal pain. What is the management?

246. Pictures from Anthology of Medical Condition: HCC, rectus sheath hematoma, RA X-ray, Burns ( page 112), BBC( pinna of ear), Colles fractures, Ulnar distal fractures, Spine fractures ( osteoporosis)

247.Pictures from Bluebook: Acanthosis nigricans, Bulemia nervosa

248. Patient with ventricular fibrillation. ECG given. MX?a. atropineb. adrenalinec. lignocaine

249. A patient with sorethroat treated with amoxicillin. After 4 days developed a rash. DX?a. Allergic reactionb. SLE

Page 71: Feb 2012 AMCQ Recall

250. A female patient presents with peripheral vascular disease. History of DM and HTN, she was smoker until 3years ago, she developed DVT syndrome and stopped smoking. DX?a. Buergers diseaseb. Raynauds phenomenonc. SLE

251. IV drug user arrives to hospital with high fever. DX?a. Rheumatic feverb. Endocarditisc. Pneumonia

Feb 18, 20121. Acute limb ischemia, what sign would require urgent surgery ?a. paresthesiab. paralysisc. rubord. swelling

2. Ct scan of Rectus sheath haematoma. Patient has increasing abdominal pain at right side. No bleeding. Patient has INR of 2.9. What isthe next management?a. observationb. drainagec. laparatomyd. laparoscopye. aspiration

3. Patient has maintenance of enoxaparin and he is schedule for a surgery. What will you give?a. Vitamin Kb. FFPc. Protamine Sulfated. Dimercaprol

4. FUNDOSCOPY PICTURE ( + microaneursym seen) : patient has a history of Diabetes Mellitus type 2 later on positive blurring of visionand sought check up. What is the likely cause?a. Diabetic retinopathyb. hypertensive retinopathyc. retinal detachmentd. macular degeneration

5. FUNDOSCOPY PICTURE ( + drusen): 78 y.o. woman had a gradual onset of bilateral visual loss. What is the diagnosis?a. CRVOb. CRAOc. Diabetic retinopathyd. Retinal detachmente. Macular degeneration

6. A 60 + y.o. man presents with cough and productive sputum, weight loss of 3 kg ( forgot the duration). He had a history of working in a gold

Page 72: Feb 2012 AMCQ Recall

mine industry and was diagnosed before of moderate silicosis. The patient is a NON-SMOKER. What is likely the diagnosis of presentsymptom?a. mesotheliomab. exacerbation of silicosisc. pulmonary tuberculosisd. bronchogenic carcinoma

7. A patient had a Subtotal thyroidectomy few hrs ago. She was noted to have stridor and difficulty of breathing. Noted swelling and redness inthe neck area. What is the immediate management?a. Call the surgeonb. remove the skin suturesc. explore the wound in the wardd. arrange intra-op exploration of the wound site

8. A picture of chest X-ray and ECG in one stem. A patient with a history of ischemic heart disease and hiatus hernia attends a dinnerconference. After eating, he suddenly developed restrosternal pain and collapse. He was rushed to ED and given oxygen and iv fluids. Whatis the next management?a.Pulmonary angiographyb. Nasogastric decompressionc. Chest tube drainaged. Emergent laparotomye. Ultrasonography( no options for ECG)

9. a 70 y.o. woman diagnosed of cancer of the caecum. What is the first symptom noted in this patient?a. palpable massb. altered bowel habitsc. tiredness and fatigued. bowel obstructione. dyspepsia

10. A 30 y.o. man with multiple rib fractures following a motor vehicle accident. He is noted of having paradoxical breathing. What would bethe most appropriate next step in management?a. strap the chest wallb. controlled and positive ventilationc. insert chest tubed. 100 % oxygen via bag mask

11. Old lady comes to the ED following MVA with a wrist fracture and confusion. What would be the most appropriate next step?a. Urgent CT headb. Xray wristc. Wrist plaster cast

12. A 55 y.o. man comes to your clinic and concerned of his family history which his father was diagnosed of having prostate cancer at 85 y.o.and neighbor was diagnosed of same condition recently. He is worried he might get prostate cancer. On DRE, his prostate was smooth, softand median sulcus was felt. No other symptoms. What would you advice?

Page 73: Feb 2012 AMCQ Recall

a. reassuranceb. PSA for the next 4 weeksc. DRE annuallyd. Ultrasound

13. Patient has an ulcer at medial malleolus of left leg. What could be the risk factor?a. smokingb. previous history of DVT at same legc. diabetes mellitusd. malignant change

14. Man had lower limb injury involving the fat layer. He had a complete vaccination history and last dose was 9 years ago. What is the mostappropriate management?a. aDTPb. aDTP and Igc. Igd. reassure

15. A 27 y.o. man had a major motor vehicular accident which liver and spleen have lacerations. multiple wounds and fractures. He received15 units of blood during resuscitation. Few hours after, it was noted positive bleeding in all sites. What is the cause?a. transfusion reactionb. hypocalcemiac. spleen ruptured. DICe. dilutional thrombocytopenia

16. A young patient had a fall injury. PO2 60 and PCO2 55. Patient has breathlessness but conscious. Whats the next step management?a. paralyze, intubate and ventilateb. 100% oxygen via bag maskc. controlled oxygend. Chest xraye. chest tube and drain

17. A patient is diabetic and was scratched by a cat in the arm and few days later, + red rash at arm, forearm and anterior shoulder ( no LAD,No fever mentioned in the stem). What is the likely diagnosis?a. Cat scratch feverb. Cellulitis

18. 18 month old boy has bilateral calf tenderness, he refused to stand. Hx of viral infection. Next step in management?a. steroidsb. immunoglobulinsc. antibiotics

19. A week old neonate was apnoeic and brought by parents to the ED. Noted of no pulse and respiration. What would be your next step?a. Inform coronerb. arrange autopsy at hospitalc. give a deatch certificate stating SIDS

Page 74: Feb 2012 AMCQ Recall

20. A 2 week old neonate with temp of 40 deg C came to hospital with poor feeding and irritability, + seizure. What is the likely diagnosis?a. febrile convulsionb. septicemiac. encephalitisd. epilepsy

21. A 10 y.o. girl is brought by her mother for severe headache associated with nausea. This occurs 3 episodes in a month. Her mother alsomentioned she had same history with the child when she was young. What is the most appropriate management?a. sumatriptanb. paracetamolc. methylsergided. ibuprofene. morphine

22. A child with a history of URTI 2 weeks ago now presents with petechiae, bruises in ant trunk and upper and lower ext. labs given: onlyplatelet count is decreased. What is the management?a. strict bed restb. gamma globulinc. steroidsd. antibiotics

23.A boy with long ears and large forehead. ( this is a long stem mentioning uncle also has same features) What is the diagnosis?a. Down syndromeb. XYYc. Klinefeltersd. Fragile X syndrome

23. What is the chance of having a child affected of cystic fibrosis if one parent is a carrier?a. 0b. 25c. 50

24. A pedigree pattern of Autosomal recessive.25. Primigravida at 28 weeks AOG, placenta low lying. and scheduled of repeat ultrasound at 34 weeks and now placenta is not low lying butseen one loop of cord around babys neck. what is the management?a. repeat ultrasound at 37 weeksb. CS at termc. tocolysisd. wait and reassuree. instrumental delivery

26. What is true regarding Bartholin cyst? ( not mentioned infected)a. can be asymptomaticb. maybe caused by gonococcal infectionc. treat with antibiotics( no option of surgical removal)

27. Patient 18 weeks AOG. What is the management to indicate premature labor?

Page 75: Feb 2012 AMCQ Recall

a. fibronectin testb. USG of cervical length

28. An RH negative woman ( G2P0) at 34 wks AOG was found to be mildly RH sensitized. Her first pregnancy was with an RH + foots. Shewas given anti-RHo after and miscarriage ensued. She had an orthopedic operation which underwent blood transfusion. Few weeks earlier,She had antibody test which was found negative. Her current pregnancy was found out the foetus is RH +. Whats the reason for patienthaving mild sensitization?a. Anti-Rho givenb. blood transfusionc. from previous pregnancyd. occult feto-maternal transfusion

29. long stem but only ask in the end: What is the most likely sign to indicate ectopic pregnancy in this patient?a. low abdominal rigidityb. profound anaemiac. Hypotension

30. 47 y.o. woman with a BMI of 30. History of amenorrhoea since 12 months. What do you expect to find in her labs?a. high estrogen and low FSHb. very low estrogen and very high FSHc. slightly low estrogen and slightly high FSHd. High estrogen and High FSH

31.A middle aged man watching tv about Australian Airforce going to Iraq. He was found boarding the plane and was held and detained.When he was asked questions, He told that the TV is telling him to go. What is this?a. Thought broadcastingb. auditory hallucination when he fall asleepc. depressed moodd. history of drug abuse

32.In Australia, psychodynamic psychotherapy is mostly recommended for which one of the following?a. schizophreniab. personality disorderc. autismd. specific phobiae. OCD

33. A young lady 16 y.o. was brought by her mother to your GP clinic complaining of her daughters behavior. She talks rudely to her father.She skips classes, school authorities plan to expel her. What is the recommended management?a. antidepressantb. cognitive behavior therapyc. family therapyd. psychotherapy

34. A lady admitted to the hospital for elective surgery. You were called because she's having conflict with the nurse. She told you that " you

Page 76: Feb 2012 AMCQ Recall

are a doctor and you understand unlike this stupid nurse". What is the diagnosis?a. Narcissistic PDb. Hypochondriasisc. Splittingd. Denial

35. A 14 y.o. aboriginal boy had history of good school performance but recently had problems with it, had auditory and visual hallucinations attimes. Hx of leaving family. What could be the cause?a. alcoholb. marijuanac. metamphetamined. petrol sniffing

36. Picture : exactly the same page 206 anthology of medical condition. what is the diagnosis?a. tinea capitisb. pityriasis capitis ( dandruff)c. pityriasis versicolord. seborrheic dermatitis

37. Husband brought his wife for check up. His wife had history of excessive cleaning the house. He also noticed his wife becomes flirtatiousto his friends and drinks alcohol. When check up by a doctor, she also flirts to the doctor and became abusive. What is likely the diagnosis?a. Histrionic personality disorderb. Hypomaniac. substance abused. Borderline personality disorder

38. Old patient has history of syncope 3 episodes. Choose ECG that fits:a. ecg of Atrial fibrillationb. ecg of Atrial flutterc. ecg of ventricular ectopic beatsd. ecg of Complete heart blocke. ecg with tall t waves ( Hyperkalaemia?)

39. Which one of the following studies is LEAST useful?a. Cohortb. case controlc. cross sectionald. case reporte. systematic review40. A picture of psoariasis at both lower legs with extensive lesions. What is the treatment of choice?a. UVB lightb. calcipotriolc. steroids( no dithranol, no methotrexate, no tar in the options)

41. A long stem about patient has dryness in the eyes and mouth. Anti RO and Anti La POSITIVE detected. Whats the management?a. Hydroxychloroquineb. Steroidsc. Methotrexate

Page 77: Feb 2012 AMCQ Recall

42. A patient was diagnosed of having colon cancer ( i forgot the stage) , surgical operation done and chemotherapy should be started. whatsthe management?a. 5Fluorouracilb. CIsplatinc. Methotrexate

43. A picture of patient having redness in the eye ( anterior uveitis). He had a history of mouth ulcer and genital ulcers. Whats the treatment?a. topical hydrocortisoneb. oral steroids44. WOF features would be absent in pseudobulbar palsy?a. dysarthriab. dysphagiac. absent gag reflex

45. A 24 y.o. lady has one episode of seizure. She experienced of having muscle jerks in her right arm few months ago. She asks you onadvice for driving?a. can drive after 6 monthsb. can drive after 3 monthsc. can drive after 12 monthscan drive after 24 months

46. Patient with a maintenance of digoxin for AF and thiazide for HTN. Recently, she complains of nausea, vomiting. and palpitations. Whatwould be the most appropriate step?a. stop digoxin and give potassium supplementsb. stop thiazide and give potassiumc. give potassium

47. Chest X-ray showing a cavitary lesion at the upper right lobe. Patient came from Sudan. He comes with fever and night sweats. Sputumnegative for acid fast bacilli. What would be the most appropriate management?a. Isoniazid prophylaxisb. Isoniazid, rifampicin, pyrazinamide and ethambutolc. Amoxicillind. Ceftriaxonee. no treatment necessary

48. A driver from interstate has moved to your locality recently and is stable on lithium for past 20 years. How du you monitor?a. check lithium every 3 monthsb. check lithium every 6 monthsc. check lithium every 1 year

49.85 y.o. woman presented with right thigh and leg pain after fall. Investigations are: calcium 3.5 ( 2.1-2.6), Urea- 10 ( phosphate level notgiven). Most common cause?a. Osteoporosisb. Vitamin D toxicityc. Primary hyperparathyroidismd. Hypercalcemia of malignancy

Page 78: Feb 2012 AMCQ Recall

50. A middle aged woman with calf pain and tenderness from few days came to hospital. Her investigations showed DVT. She has no otherassociated symptoms. What is the management?a. inpatient management with compression stocking and LMWHb. IP management with elevation and LMWHc. outpatient management with compression and LMWHd. OP management with leg elevation and LMWH

51. Middle aged man complains of cough and productive sputum. CXR showed patchy infiltrates. What is the management?a. Oral amoxicillin and clavulanateb. Iv ampicillinc. oral flucloxacillind. Oral azithromycine. tetracycline

52. Young girl collapsed in a party brought to ED. She is confused. fever of 40 deg C. Noted muscle jerks. BP= 157/98. What is the initialmanagement?a. Thiamine and oral hydrationb. cold saline infusion and cold blanketc. diazepamd. haloperidol

53. ALL are side effect of steroids EXCEPT:a. hirsutismb. ecchymosisc. osteomalaciad. lymphopeniae. increase WCC

54. 65 y.o. man had a headache with tenderness on the scalp esp on the right side. Positive history of headaches for few days. What is thenext management?a. temporal biopsyb. oral steroidsc. CT scan( no ESR or FBC options)

55. In a patient with GBS which is appropriate way to monitor his oxygenation?a. forced vital capacityb. ABGc. pulse oximeterd. O2 saturation

56. A man complains of cough productive with green sputum and occasional blood. No fever. WOF is the best long term management?a. low dose tetracycline for 12 monthsb. anti tb drugsc. postural drainaged. reassure

57. A 38 y.o. woman presents with secondary amenorrhoea for 12 months. her blood results showing FSH increased and Oestradiol

Page 79: Feb 2012 AMCQ Recall

decreased, prolactin normal and TSH normal. USG has 3-4 cysts in the ovary. What is the diagnosis?a. Premature menopause ( exact option not written as premature ovarian failure)b. pituitary adenomac. PCOSd. pregnancy

58. A patient had history of surgery and noted palpable mass in midline.a. cough when lyingb. cough when sittingc. ask patient to voluntary raise handsd. passive raise patients legs

59. Patient has a right LAD when biopsy + squamous cell ca. Where is the primary site?a. breastb. thyroidc. bronchusd. kidney

60. Toxic shock syndrome caused by staphylococcus aureus is associated with:a. super absorbent sanitary padsb. endotoxin production

61. Construction worker complains of eye redness and lacrimation after hammering a nail at work. + metal spikes seen. Whats is yourmanagement?a. antibioticsb. irrigation of fluidsc. local anesthetic and probe the eye

62. an indigenous boy complains of greenish discharge coming out from the ear and also has green nasal secretions. What is the appropriatemanagement?a. swab ear dischargeb. swab nasal dischargec. audiometry( no option antibiotic)

63. An alcoholic man with pain radiating to the back. Diagnosed of acute pancreatitis. WOF is the LEAST likely to present with?a. constant painb. nocturiac. diarrhoead. regurgitation of food

64.A patient complains of lower back pain. On PE noted lumbar spine is not in its normal curvature. NO numbness or decrease ankle jerk.What to do next?a. lumbosacral xrayb. MRIc. Nerve conduction study

65. A patient came in with haematuria, dysuria. Urinalysis showed many rbi and atypical nuclei. What to do next?a. renal ultrasound

Page 80: Feb 2012 AMCQ Recall

b. ct scan of abdomenc. urine culture and sensitivity

66. A man with scrotal swelling noted palpable cord and is thickened on PE. What is your investigation?a. ultrasoundb. ct scanc. FNACd. AFP

67. An elderly patient brought to ED with lightheadedness and palpitations. was diagnosed of Aortic stenosis. ECG is given. What is themanagement?a. adenosineb. digoxinc. verapamild. not to repeat the carotid massage68. A 39 y.o. woman came to see you, and asking for HPV vaccination. What will you advice? ( this is longer stem)a. HPV is not indicatedb. Give HPV vaccine now

69. An 18 month old infant has in toeing. what will you advice the mother?a. reassureb. refer to ortho

70. 5 month old infant had constipation. Started solid foods given 3x a week. still continuous breastfeeding. What is the cause of constipation?a. inadequate fluid intakeb. starting solids

71. A primigravida presented at term for delivery, IE during admission: 1 cm cervical dilatation, 30-40 % effaced, no rupture of membranes,station 0. After 4 hrs, IE done with 4 cm dilated cervix, fully effaced and station +2. Whats the next step?a. amniotomyb. oxytocin infusionc. wait another 4 hrs and assessd. CS

72. A 42 y.o. Nulliparous woman has a bulging mass in the perineum especially during straining on defecation. What is the likely cause?a. cystocoeleb. rectocoele

73. A pregnant woman ( forgot the AOG, its on 2nd or 3rd tri, 24 wks or 30 plus) has cervical ectropion. What is the management?a. Pap smear nowb. Pap smear 4 weeks postpartumc. do nothing

74. A woman ( forgot the age) was diagnosed of having CIN II. whats the most appropriate management?a. Cone biopsyb. LEEP

Page 81: Feb 2012 AMCQ Recall

c. reassured. hysterectomy

75. Infant with 2 day history of cough and fever temp of 37.8 deg C, wheezing heard. Child is irritable, + episodes of vomiting. WOF is thenext step in management?a. IV fluidsb. Salbutamol nebulisationc.Observed. antibiotics

76. A mother brings her 5 y.o. girl who has sore down below. On PE, the vulva is reddened and no other signs. What is your management?a. report to child protection authorityb. examine under general anesthesiac. avoid bubble bathd. full blood count

77. A 38 y.o. man was attending a party and ate some peanuts in the cake, suddenly had hoarseness of voice and difficulty of breathing.Whats the immediate management?a. Adrenaline IMb. intubate and ventilatec. Antihistamined. administer Oxygen

78. A child had a URTI last 2 weeks ago. Her mother noticed of few bruises and petechiae. No other abnormalities seen, PE is normal. Whatis the next step?a. FBCb. ANAc. Urine microscopy

79. A 10 y.o. boy has fever and knee pain at first then later on, has ankle pain. Few rashes on the back. What is the management?a. Penicillinb. IV immunoglobulinc. steroids

80. A 6 month old infant presents with lethargy, poor feeding, No fever, tachypnoea ( this is a long stem forgot other infos). What is the nextinvestigation?a. ABGb. electrolytesc. chest xray

81. A 16 y.o. girl came to see her GP. She finds it difficult to concentrate for 6 months and previously did well in academic studies. Herappetite hasn't change. She prefers to be alone in her room most of the time when she's at home. She goes with her friends but she finds itvery hard to entertain herself. Probable diagnosis?a. Major depressionb. normal adolescent behaviorc. prodromal schizophreniad. Dysthymic diorder

Page 82: Feb 2012 AMCQ Recall

82. An 89 y.o. woman living in a nursing home is found masturbating in the privacy of her room by the nurse. The nurse asks you to intervene.What is the best step?a. refer for counsellingb. do nothing as its her private affairc. tell the patient that his behavior is inappropriate

83. An 83 y.o. old woman living in a rest home complains of insomnia. On asking she says, she is not depressed and does not suffer fromany other illness and is not on any medication. What is your next step?a. Sleep hygieneb. antidepressantc. relaxation therapy and meditation

84. A colleague fed up with work told you he drinks to fall asleep. What to do?a. tell head of the internb. Hospital directorc. Advice him to seek professional adviced. Tell medical board

85. A young man is agitated and presents to ED with a knife in his hand. He has been seeing a psychiatrist in the past and refuses to give anydetails regarding his illness. Later on, patient is calm and handed you the knife. What will you do next?a. restrain himb. talk to him regarding his psychiatric illnessc. talk to the psychiatrist against patients wishd. ask his relative about his mental history

86. Young man lost his job recently broke up with girlfriend. History of drug abuse and violent behavior. WOF is he likely to benefit from? ( thestem is much longer than this)a. Behavior therapyb. motivational therapyc. psychotherapy

87. A 50 y.o. woman complains of pain at maxillary area and some redness. ( no rashes mentioned). What is the appropriate management?a. Carbamazepineb. Acyclovirc. Famciclovir

88. A study testing drug B reveals significant benefit of drug B> drug A with probability of < 0.01. this means:a. Drug A is better than Bb. Drug B is better than Ac. The difference between drug A and drug B occur by chance ( ? not the exact option)

89. A GP is in rural area, you notice that recently more cases of Hepatitis C has been diagnosed and want to do research to find out theincidence of hepatitis C in the community with a population of around 3000. How will you get the information you need for the research?a. Find the number of all patients with current hepatitis C antibodyb. All patients with current hepatitis C PCR positivec. All patients with current Hepatitis C antigen positive

Page 83: Feb 2012 AMCQ Recall

d. All cases diagnosed as hepatitis C positive for the past 5 years

90. A man with heavy alcohol intake brought to ED. BP 180/100 HR 120, + tremors ( fine) bilateral in both hands. No confusion, not agitated.What is the appropriate management?a. Propanololb. Benztropinec. Diazepamd. Haloperidol

91.Naltrexone is:a. short actingb. easily absorbed orallyc. substitute for methadone in treating drug dependence

92. A case of 45 y.o. woman having menorrhagia. was diagnosed of intramural fibroids, has enlarged uterus. What is the management?a. low dose Oral contraceptive pillb. Oral oestrogen and progesterone hormone replacement therapyc. norethisteroned. Depo provera

93. A picture of CTG ( looks like early deceleration). What is the next management?a. CSb. Amniotomyc. ultrasoundd. repeat CTG after 24 He. continuous CTG monitoring

Extra1.17 years old jogging then collapsed.somebody did CPR and was revived when she arrived atER. Has history that father died suddemly. Which is the likely diagnosis?--HOCM2. ECG on V tach. Identify and Treatment--Amiodarone3. Woman when she defecates a mass is noted to bulge out of her introitus.Most likelydiagnosis.---Rectocoele4. woman with increased LH and FSh. Most likely diagnosis---ovarian tumor--pitiutary tumor5. child with constipation. What is the next management--laxative6. woman with hot flushes. Had hysterectomy years ago. What isthe most appropriatetreatment?--conjugated equine estrogen 0.6257. child with bruises and petechiae after a viral infection weeks ago.most likely diagnosis?--ITP8. ct scan of rectus sheath hematoma9. july 2010 question number 5310. situation about testicular sweeling. What do you do next?--ultrasound--FNAC

Page 84: Feb 2012 AMCQ Recall

11. question about painless hematuria--bladder tumor--renal tumor12. question about a man 35years old asking about his risk of developing prostate CA. Fatherdiagnosed prosteate CA at 85 years old.--reassure?--FOBT--colonoscopy13. what is the most important sign to indate for immediate embolectomy--paralysis--pain--rubor14. 6weeks old infant with fever. Mother notes some twitches before. What is the most likelydiagnosis--septiceamia--febrile convulsion15. old man was found by police and brought to hospital. Daugther has many complaints aboutfather. She said he has been living in squalor and eating rubbish.what kind of dementia?--frontal lobe dementia--lewy body dementia16. what to give to pregnant woman aside from folic acid to prevent NTD?--iodine17. question july 2010 #10118. question july 2010 #11019. question july 2010 #11120. question about an old lady after closing the window (?) sustained a crush fracture on hervertebrae. She had a history of opertation for CA of ceucum--metastasis from primary cancer--osteoporosis21. patient just had cricothyroidectomy and developed stridor while she was still in therecovery room. What will you do?--call the surgeon--explore the wound in the ward--remove the skin sutures22. xray given. Man is a non-smoker and working in the mine industry (did mention the years,think it was more than 20years). Has been coughing--mesothelioma23. question about percentage passing the disease to their children with cystic firbosis-- 0%24. xray given. Man coughing, smoker and working in the mine industry. What is the mostprobable diagnosis?--brochogenic CA25. a child with fever. What is the indication that she has a severe bacterial infection--unresponvie to mother--pallor26. xray of sigmoid volvulus27. Boy having balck and white drawings. Parent just separted. What is the most appropriateMx?--ask the boy about his drawing28. question about domestic violence. Pregnant lady with bruises. What do you do?-- give the numbers29. woman with 3 children wants space, but in file said “do not give OCP” as Stand byhusband-- its her opinion and give her OCP

Page 85: Feb 2012 AMCQ Recall

30. NALOXONE is-- Short acting31. 17yo wants OCP. She had seizsure whihc is controlled with carbamepine and subduralhaematoma-- GIVE OCP--GIVE OCP AND USE CONDOM*• X-ray of hip osteopenia ALP is 800 ( that is the only information given ) Rx• Biphosephanate• Ca• Vita b12• Radiotherapy• Pic of both lwoer limbs showing red rash. 34y old male with diarrhoea and admitted inthe hospital Dx• HSP• Mallert finger pic. Which type of injury will occur• Hyperflexion occur due to extension• Rupture of tendon• CXR – night sweat, dry cough, wt loss, enlarge bilateral cerviacal lymphnode Dx• Sacodosis• Tb• Lymphoma• CLL• A patient who started hydrochlorothiazide with digoxin develop nausea electrolyte picgiven only potassium decrease• Stop digoxin and give K suppliment• Stop hydrochlorothazide and give K supp• SVT ECG• Adenosing• Amiodarone• Verapermil• Dogoxin• ECG . a pt who had stroke came to hospital on 5# day she collapsed in the toilet. BPpulse?? What will you do next to dx his condition?• Ctpa• Echo cardio• A boy came with bloody diarrhea with less urine output what is the dx• Renal failure• HUS• A infected batholian cyst• Antibiotic will resolve it• Cause by gonnorrhoea• It is asymptomatic• A pt who had aninjury to eye due to hit from metalic spike what will you do• Use local anaesthetic and removed• A girl comes with 6 week ammenorrhoea diagnosed she is preg. After home preg test.Now develop lower abdominal pain and slight vaginal discharge• Quntitative serial BHCG• USG• P/V exam• A prenant lady 22 week developed lower abd pain tenerness in the left iliac fossa andsupra pubic region. Appendicitemy done when she is tennager. Dx• Incidental finding of red degen of ut fibroid• Round lig pain.• A pt 70 yrs old man came to the hospital think that he is having serial illness coming todoctor with different names. What is it

Page 86: Feb 2012 AMCQ Recall

• Facticious dis• Depression• Normal grief• Abd x-ray 5thPOD due to hip replacement• Pt rectus tube• Gastro graffin enama• Colonscopy• 5th POD for input output chart given.• In both recall output is more than input. One is 1400 is differnt, other recall 2100.• Resolved paralytic ilius• Wrong chart• A lady and husband came to you recently dx cystic fibrosis concern abouthe their childgetting it• 0.25 chance• 0.5 chance• A couple comes telling that one child diagnosed as cystic fibrosis concern about nextchild incidence• 0.5• 0.25• 100• You went to old house to trat a person and prescribe b-blocker and asprin. The nursetold that he is not responding to B-blocker what ill you do• Talk with nurse durin her break• Report nurse manager• Do as she told• You consider to old care notice that it is written in their chart telling dont crash themedicine. But nurse use to crush the medicine to pt as it is hard for him to swallowingwhat will you do• Complain nurse manager• Go to nurse board• Complain to the sup in age care.• Ovarian Ca where is the primary site?• Breast• Lung• CT head show hemorrhage. Pt on warfarrin and INR 4.5 what will you do• giveFFP• give platelet• CT head showing same lesion like infaction• Multi infarct dementia• Cerebral hmg• Cerebral abcess• A prg lady came to with you are 18weeks. Her last preg is a premature delivery what tokonw the outcome his• Cervical length• Fibronectin• VT ecg• Adenosin• Amiodaerone• Adrenaline• A lady comeplain loss of finger grip in index and thumb what nerve is having.• Radial n.• Carpal tunnel• Ant. Intoressisis• Ulner n.

Page 87: Feb 2012 AMCQ Recall

• 80 lady comeing with rectocele• Pessary• Sx• Pelvic exercise• 50 old man coming with sccrotal pain which is reddened what ill you do• Urine catha• b/d culture• FNAC• 50 male taken to a research and during it he become dementia and his primary carier ishis son what will you do• Ask from son• Gardianship• Ethic and comity• Fundoscopy pic, HTN ployuria, ploydypsia, what is dx• DM• HTN• CRAO• CRVO• A male who use to work in mine in 20yr smoker 40yr what is the dx. CRX given• Smallcell carcinoma• Squamous cell carcinoma• Metho• A chinese lady come to your. Husband is recenly dx as hemochromatosis having 9 &18 year kid. Wife is not screen for heamochromtosis. What will you do• Check wife only• Check 9 yr only• Check 18 year• Check both children• A down syndrom under go surgery develop hypersensitive• Latex• Anaestheic drugs• A 34yr male havin, ACEI Statin b-blocker develop calf pain tenderness in calf• Check ck level creatin• Check FBC• A pic both eye increase tearing who will you do• Refer plastic surgery• Cold compression• CXR given and ask of diagnosis• Pul edema• Metastasis• Carcinoma• A 2nd POD a person develop agitated confusion. Temp normal and pulse and BPnormal what is the cause• Alcohol withdrawal• Pulmonary atelectesia• Electrolyte imbalance• A yong boy comes with rashes in both hand and arthralgia. What will you do to dx it• CRP and ESR• Echo• CXR•

All The best ……………………………….